Крок 1 - Стоматологія 2010 (буклет)

1 / 200
У гетерозиготних батьків з A(II) і B(III) групами крові за системою АВ0 народилась дитина. Яка імовірність наявності у неї 0(I) групи крові? A child was born to heterozygous parents with A(II) and B(III) blood groups according to the AB0 system. What is the probability that she has 0(I) blood group?

75% 75%

50% 50%

0% 0%

100% 100%

25% 25%

2 / 200
У мешканців Закарпаття внаслідок дефіциту йоду в харчових продуктах часто зустрічається ендемічний зоб. Який вид мінливості спричиняє це захворювання? Inhabitants of Transcarpathia often have endemic goiter due to iodine deficiency in food. What kind of variability causes this disease?

Комбінативна Combinative

Мутаційна Mutational

Онтогенетична Ontogenetic

Співвідносна Relative

Модифікаційна Modifying

3 / 200
У медико-генетичну консультацію звернулася хвора дівчинка з попереднім діагнозом 'синдром Шерешевського-Тернера'. Яким генетичним методом можна уточнити діагноз? A sick girl with a preliminary diagnosis of 'Shereshevsky-Turner syndrome' turned to medical and genetic counseling. What genetic method can be used to clarify the diagnosis?

Дерматогліфіка Dermatoglyphics

Біохімічний Biochemical

Визначення статевого хроматину Definition of sex chromatin

Генеалогічний Genealogical

Гібридологічний Hybridological

4 / 200
Провідником наукової експедиції по Індії був місцевий житель, який ніколи не розлучався зі своїм улюбленим собакою. Якими інвазійними захворюваннями можуть бути заражені члени експедиції при контакті з цим собакою, якщо він є джерелом інвазії? The guide of the scientific expedition in India was a local resident who never parted with his beloved dog. What invasive diseases can the members of the expedition be infected by contact with this dog, if is it a source of infection?

Ехінококоз Echinococcosis

Фасціольоз Fasciolosis

Дикроцеліоз Dicroceliosis

Теніоз Taeniosis

Парагонімоз Paragonimosis

5 / 200
У клітині людини в гранулярну ендоплазматичну сітку до рибосом доставлена і-РНК, що містить як екзонні, так і інтронні ділянки. Який процес НЕ ВІДБУВАЄТЬСЯ? In a human cell, i-RNA containing both exonic and intronic regions is delivered to the granular endoplasmic reticulum to ribosomes. Which process does NOT occur?

Трансляція Broadcast

Транскрипція Transcription

Процесінг Processing

Пролонгація Extension

Реплікація Replication

6 / 200
Через два тижні після переливання крові у реципієнта виникла лихоманка. Про яке протозойне захворювання повинен думати лікар? Two weeks after the blood transfusion, the recipient developed a fever. What protozoan disease should the doctor think about?

Лейшманіоз Leishmaniasis

Малярія Malaria

Токсоплазмоз Toxoplasmosis

Амебіаз Amebiasis

Трипаносомоз Trypanosomosis

7 / 200
Під час огляду ротової порожнини встановлено, що у пацієнта має місце дотикання ріжучих країв верхніх та нижніх різців. Для якого прикусу характерне таке розміщення зубів? During the examination of the oral cavity, it was found that the cutting edges of the upper and lower incisors are touching in the patient. What bite is characterized by such an arrangement of teeth?

Прогенія Progeny

Прямий прикус Direct bite

Біпрогнатія Biprognathia

Ортогнатія Orthognathia

Закритий прикус Closed Bite

8 / 200
У хворого інфаркт міокарда в ділянці передньої стінки лівого шлуночка. В басейні якої судини виникло порушення кровообігу? The patient had a myocardial infarction in the area of the front wall of the left ventricle. In which vessel's basin did the circulatory disturbance occur?

Передня шлуночкова гілка правої вінцевої артерії Anterior ventricular branch of the right coronary artery

Передсердно-шлуночкова гілка лівої вінцевої артерії Atrioventricular branch of the left coronary artery

Ліва крайова гілка лівої вінцевої артерії Left marginal branch of the left coronary artery

Огинаюча гілка лівої вінцевої артерії Enveloping branch of the left coronary artery

Передня міжшлуночкова гілка лівої вінцевої артерії Anterior interventricular branch of the left coronary artery

9 / 200
Після травми хворий не може розігнути руку у ліктьовому суглобі. Порушення функції якого основного м’язу може це спричинити? After the injury, the patient cannot extend the arm at the elbow joint. What major muscle dysfunction can cause this?

Musculus levator scapule Musculus levator scapule

Musculus teres major Musculus teres major

Musculus subscapularis Musculus subscapularis

Musculus infraspinatus Musculus infraspinatus

Musculus triceps brachii Musculus triceps brachii

10 / 200
Яка артерія може бути пошкоджена при виконанні провідникової анестезії в ділянці отвору нижньої щелепи? Which artery can be damaged during conduction anesthesia in the area of the opening of the lower jaw?

Нижня альвеолярна артерія Inferior alveolar artery

Язикова артерія Lingual artery

Середня оболонкова артерія Medium sheath artery

Щічна артерія Buccal artery

Крилоподібні гілки Winged branches

11 / 200
Чоловік 65-ти років звернувся до стоматолога зі скаргою, що в нього нижня щелепа не рухається назад. Встановлено, що в нього після падіння пошкоджений м’яз: A 65-year-old man went to the dentist with a complaint that his lower jaw does not move back. It was determined that he had a damaged muscle after a fall:

Жувальний Chewing

Присередній крилоподібний Medium pterygium

Скроневий Temporal

Бічний крилоподібний Lateral pterygoid

Двочеревцевий Bigastric

12 / 200
У хворого після застудного захворювання з’явилося порушення больової і температурної чутливості передніх 2/3 язика? Який із нервів при цьому постраждав? After a cold, the patient developed a violation of pain and temperature sensitivity of the front 2/3 of the tongue? Which of the nerves was affected?

Під’язиковий Sublingual

Трійчастий Triple

Язикоглотковий Lingopharyngeal

Додатковий Additional

Блукаючий Wandering

13 / 200
У хворого кривошия. Який м’яз шиї уражений? The patient has a torticollis. Which neck muscle is affected?

M.omohyoideus M.omohyoideus

M.mylohyoideus M.mylohyoideus

M.sternohyoideus M.sternohyoideus

M.sternocleidomastoideus M.sternocleidomastoideus

M.platisma M.platisma

14 / 200
У хворого 25-ти років, який скаржився на утруднене ковтання, виявлене пухлиноподібне підвищення на корені язика 1-2 см в діаметрі, в ділянці сліпого отвору. Розростання залишків якої залози встановив лікар? A 25-year-old patient who complained of difficulty swallowing was found to have a tumor-like growth 1-2 cm in diameter at the root of the tongue, in the area of the blind hole. Residual growth which gland did the doctor install?

Прищитоподібна Pythroid

Під’язикова Sublingual

Щитоподібна Thyroid

Аденогіпофіз Adenohypophysis

Вилочкова Vylochkova

15 / 200
Хворий скаржиться на біль під час жування, особливо під час висунення нижньої щелепи і зміщення її вбік. Функція яких м’язів порушена? The patient complains of pain during chewing, especially when extending the lower jaw and moving it to the side. Which muscles function is impaired?

Медіальні крилоподібні Medial pterygoids

Латеральні крилоподібні Lateral Pterosaurs

Скроневі Temporal

Щелепно-під’язикові Maxillohyoid

Жувальні Chewing

16 / 200
Під час виконання операції з приводу пухлини черевної частини сечоводу у хворого, лікар повинен пам’ятати, що попереду його розташована важлива артеріальна судина. Яка саме? When performing an operation for a tumor of the abdominal part of the ureter in a patient, the doctor must remember that an important arterial vessel is located in front of him. Which one exactly?

a. iliaca communis a. iliaca communis

a. testicularis a. testicularis

a. renalis a. renalis

a. iliaca interna a. iliaca interna

a. ileocolica a. ileocolica

17 / 200
Після падіння з дерева у хлопчика утруднене відведення руки до горизонтального положення. Який із м’язів імовірно пошкоджений? After falling from a tree, the boy has difficulty moving his arm to a horizontal position. Which of the muscles is probably damaged?

M.deltoideus M.deltoideus

M.triceps brachii M.triceps brachii

M.supinator M.supinator

M.anconeus M.anconeus

M.coracobrachialis M.coracobrachialis

18 / 200
Під час оперативного втручання у жінки виникла необхідність перев’язати маткову артерію. Яке з утворень може бути випадково перев’язаним разом з нею? During surgery, the woman had to ligate the uterine artery. Which of the formations can be accidentally ligated together with it?

Маткова труба Uterine tube

Кругла зв’язка матки Round ligament of the uterus

Внутрішня клубова вена Internal iliac vein

Сечівник Urine

Сечовід Ureter

19 / 200
При аускультації серця був визначений діастолічний шум в II міжреберному проміжку по правій пригруднинній лінії. Про патологію якого клапана це свідчить? During auscultation of the heart, a diastolic murmur was detected in the II intercostal space along the right parasternal line. Which valve pathology does this indicate?

Клапан легеневого стовбура Pulmonary trunk valve

- -

Двостулковий Bifold

Клапан аорти Aortic valve

Тристулковий Triple

20 / 200
У хворої з пухлиною підшлункової залози розвинулася механічна жовтяниця внаслідок стиснення жовчовивідної протоки. Яка протока піддається стисненню? A patient with a pancreatic tumor developed mechanical jaundice as a result of compression of the bile duct. Which duct is subject to compression?

Ductus hepaticus communis Ductus hepaticus communis

Ductus choledochus Ductus choledochus

Ductus cysticus Ductus cysticus

Ductus hepaticus sinister Ductus hepaticus sinister

Ductus hepaticus dexter Ductus hepaticus dexter

21 / 200
У гістологічному препараті шліфа зуба у міжклітинній речовині визначаються колагенові волокна, що йдуть тангенційно до дентино-емалевої межі і перпендикулярно до дентинних трубочок (волокна Ебнера). Назвіть даний шар дентину: In the histological preparation of a tooth slide, collagen fibers are identified in the intercellular substance, which go tangentially to the dentin-enamel border and perpendicular to the dentin tubules (Ebner's fibers). Name this layer dentine:

Навколопульпарний дентин Peripulpal dentine

Вторинний дентин Secondary dentin

Інтерглобулярний дентин Interglobular dentine

Плащовий дентин Coat dentin

Зернистий шар Grain layer

22 / 200
Хворій 35-ти років з діагнозом безпліддя в гінекологічному відділенні зроблено діагностичну біопсію ендометрію. При мікроскопічному дослідженні з’ясувалося, що слизова оболонка з явищами набряку, маткові залози звивисті, заповнені густим секретом. Надлишок якого гормону обумовлює такі зміни в ендометрії? A 35-year-old patient with a diagnosis of infertility underwent a diagnostic biopsy of the endometrium in the gynecological department. Microscopic examination revealed that the mucous membrane is swollen, the uterine glands are tortuous , filled with a thick secretion. The excess of which hormone causes such changes in the endometrium?

Прогестерон Progesterone

Естроген Estrogen

Тестостерон Testosterone

АКТГ ACTH

Соматотропін Somatotropin

23 / 200
У крові дівчини 16-ти років, котра страждає на аутоімунне запалення щитоподібної залози, виявлено численні плазматичні клітини. З проліферацією та диференціюванням яких клітин крові пов’язано збільшення кількості плазмоцитів? In the blood of a 16-year-old girl suffering from autoimmune inflammation of the thyroid gland, numerous plasma cells were detected. The proliferation and differentiation of which blood cells is associated with an increase in the number plasma cells?

Т-кілерів T-killers

В-лімфоцитів B-lymphocytes

Т-супресорів T-suppressors

Тканинних базофілів Tissue basophils

Т-хелперів T-helpers

24 / 200
У хворого при гастроскопії виявлено недостатню кількість слизу, що вкриває слизову оболонку. З порушенням функції яких клітин стінки шлунка це пов’язано? The patient's gastroscopy revealed an insufficient amount of mucus covering the mucous membrane. What cells of the stomach wall are affected?

Головні екзокриноцити залоз шлунка Main exocrinocytes of the stomach glands

Клітини призматичного залозистого епітелію Prismatic glandular epithelium cells

Ендокриноцити Endocrinocytes

Парієтальні клітини залоз шлунка Parietal cells of gastric glands

Шийкові клітини залоз шлунка Cervical cells of gastric glands

25 / 200
В червоному кістковому мозку в постембріональному гемопоезі в клітинах одного з диферонів поступово знижується базофілія цитоплазми і підвищується оксифілія, ядро виштовхується. Для якого виду гемопоезу характерні дані морфологічні зміни? In the red bone marrow during postembryonic hematopoiesis in the cells of one of the diferons, cytoplasmic basophilia gradually decreases and oxyphilia increases, the nucleus is pushed out. What type of hematopoiesis are these morphological changes characteristic of?

Лімфопоез Lymphopoiesis

Нейтрофілоцитопоез Neutrophilocytopoiesis

Базофілоцитопоез Basophyllocytopoiesis

Еритропоез Erythropoiesis

Еозинофілоцитопоез Eosinophilocytopoiesis

26 / 200
Недорозвиненість яких відділів лицьового черепу в ембріональний період призводить до появи такої вади розвитку, як 'вовча паща'? Underdevelopment of which parts of the facial skull in the embryonic period leads to the appearance of such a malformation as 'wolf mouth'?

Нижньощелепні відростки Mandibular processes

Нижньощелепні та піднебінні відростки Mandibular and palatine processes

Лобні відростки Frontal processes

Лобні та верхньощелепні відростки Frontal and maxillary processes

Піднебінні відростки Palates

27 / 200
Під час розвитку облітеруючого атеросклерозу у хворих виникають зміни в судинах нижніх кінцівок. На гістологічному препараті такої судини добре виражені внутрішня та зовнішня еластичні мембрани, у середній оболонці багато міоцитів. Яка судина ушкоджується при цьому захворюванні? During the development of obliterating atherosclerosis in patients, changes occur in the vessels of the lower extremities. The internal and external elastic membranes are well defined on the histological preparation of such a vessel, and there are many myocytes in the middle membrane. Which vessel is damaged in this disease?

Вена з сильним розвитком м’язів A vein with strong muscle development

Артерія змішаного типу Mixed type artery

Артерія м’язового типу Muscular artery

Лімфатична судина Lymphatic vessel

Артерія еластичного типу Artery of elastic type

28 / 200
У порожнині матки було виявлене плідне яйце, не прикріплене до ендометрію. Якій стадії розвитку відповідає зародок? A fertile egg was found in the uterine cavity, not attached to the endometrium. What stage of development does the embryo correspond to?

Нейрула Neurula

Гаструла Gastrula

Бластоциста Blastocyst

Зигота Zygote

Морула Morula

29 / 200
На гістологічному препараті очного яблука видно структуру, що має вигляд двоопуклого утвору, сполученого з циліарним тілом за допомогою волокон війкового пояска, зверху вкритий прозорою капсулою. Назвіть цю структуру: On the histological preparation of the eyeball, a structure can be seen that has the appearance of a biconvex formation, connected to the ciliary body with the help of the fibers of the ciliary belt, and covered by a transparent capsule on top. Name this structure:

Війкове тіло Ciliary body

Рогівка Cornea

Кришталик Crystal

Скловидне тіло Vitreous body

Склера Sclera

30 / 200
У жінки 30-ти років зменшений вміст ферментів у підшлунковому соці. Недостатня секреція якого гастроінтестинального гормону може бути причиною цього? A 30-year-old woman has a reduced content of enzymes in the pancreatic juice. Insufficient secretion of which gastrointestinal hormone can be the cause of this?

Холецистокінін-панкреозимін Cholecystokinin-pancreozymin

Секретин Secretin

Вазо-інтестинальний пептид Vaso-intestinal peptide

Шлунково-інгібуючий пептид Gastric inhibitory peptide

Соматостатин Somatostatin

31 / 200
Відомо, що у людей, які постійно мешкають в умовах високогір’я, збільшується вміст еритроцитів в одиниці об’єму крові. Це сприяє оптимальному виконанню кров’ю такої функції: It is known that the content of erythrocytes per unit volume of blood increases in people who live permanently in the highlands. This contributes to the optimal performance of such functions:

Транспортування амінокислот Amino acid transport

Транспортування газів Gas transportation

Пвдтримка іонної рiвноваги Maintenance of ion balance

Підтримка кислотно-лужної рiвнова-ги Maintenance of acid-alkaline balance

Участь у гемостазi Participation in hemostasis

32 / 200
У ході тренування на велоергометрі спортсмен підбирав навантаження для досягнення максимальної величини роботи, що виконується його м’язами. Якою у даному випадку має бути величина навантаження на м’язи спортсмена? During training on a cycle ergometer, the athlete selected the load to achieve the maximum amount of work performed by his muscles. What should be the amount of load on the muscles in this case an athlete?

Тривала мінімальна Duration minimum

Середня Average

Максимальна Maximum

Чергування мінімального і максимального Alternating minimum and maximum

Мінімальна Minimal

33 / 200
Людина стоїть у кімнаті в легкому одязі; температура повітря +14o C. Вікна і двері зачинені. Яким шляхом вона віддає найбільше тепла? A person is standing in a room in light clothes; the air temperature is +14o C. The windows and doors are closed. Which way does he give off the most heat?

Конвекція Convection

Теплопроведення Heat conduction

Теплорадіація Thermal radiation

Перспірація Perspiration

Випаровування Evaporation

34 / 200
При аналізі ЕКГ людини з’ясовано, що у другому стандартному відведенні від кінцівок зубці T позитивні, їх амплітуда та тривалість у нормі. Вірним є висновок, що у шлуночках серця нормально відбувається процес: When analyzing a human ECG, it was found that in the second standard lead from the limbs T waves are positive, their amplitude and duration are normal. The conclusion is correct that in the ventricles the process of the heart is normal:

Деполяризації Depolarizations

Реполяризації Repolarizations

Скорочення Abbreviation

Розслаблення Relax

Збудження Arousal

35 / 200
У хворого 70-ти років діагностовано крововилив у стовбур мозку. Обстеження виявило підвищення тонусу м’язів-згиначів на тлі зниження тонусу м’язів-розгиначів. Подразненням яких структур мозку можна пояснити зміни тонусу м’язів? A 70-year-old patient was diagnosed with a brain stem hemorrhage. The examination revealed an increase in the tone of the flexor muscles on the background of a decrease in the tone of the extensor muscles. The irritation of which can brain structures explain changes in muscle tone?

Чотиригорбикова структура Four hump structure

Червоні ядра Red kernels

Ретикулярна формація Reticular Formation

Чорна речовина Black Matter

Вестибулярні ядра Vestibular nuclei

36 / 200
Людина, що приймає блокатор мембранних циторецепторів синапсів еферентних провідників автономної нервової системи, скаржиться на сухість у роті. Які з рецепторів у неї заблоковані? A person taking a blocker of membrane cytoreceptors of synapses of efferent conductors of the autonomic nervous system complains of dry mouth. Which receptors are blocked?

α-адренорецептори α-adrenoceptors

Н-холінорецептори H-cholinergic receptors

M-холінорецептори M-cholinergic receptors

Н2-рецептори H2-receptors

β-адренорецептори β-adrenoceptors

37 / 200
У дитини наявне порушення формування емалі та дентину зубів через знижений вміст іонів кальцію в крові. Дефіцит якого гормону може спричинити такі порушення? The child has a violation of the formation of enamel and dentin of the teeth due to a reduced content of calcium ions in the blood. A deficiency of which hormone can cause such violations?

Паратгормон Parathyroid hormone

Соматотропний гормон Somatotropic hormone

Тирокальцитонін Tyrocalcitonin

Трийодтиронін Triiodothyronine

Тироксин Thyroxine

38 / 200
У хворого на ЕКГ виявлено, що інтервал RR дорівнює 1,5 с, частота серцевих скорочень - 40/хв. Що є водієм ритму серця? The patient's ECG revealed that the RR interval is 1.5 s, the heart rate is 40/min. What is the driver of the heart rhythm?

Ліва ніжка пучка Гіса Left leg of His bundle

Синусовий вузол Sine Node

Права ніжка пучка Гіса Right leg of His bundle

Пучок Гіса His Bundle

Атріовентрикулярний вузол Atrioventricular node

39 / 200
При обстежені пацієнта встановили що він має сильний, врівноважений, інертний тип вищої нервової діяльності за Павловим. Який темперамент за Гіппократом має цей пацієнт? When examining the patient, it was established that he has a strong, balanced, inert type of higher nervous activity according to Pavlov. What temperament does this patient have according to Hippocrates?

Флегматичний Phlegmatic

Меланхолічний Melancholic

Холеричний Choleric

- -

Сангвінічний Sanguine

40 / 200
У людини вміст глюкози в крові 15 ммоль/л (поріг реабсорбції - 10 ммоль/л). Наслідком цього буде: A person has a blood glucose level of 15 mmol/l (the reabsorption threshold is 10 mmol/l). The result will be:

Зменшення секреції вазопресину Decreased vasopressin secretion

Зменшення діурезу Diuresis reduction

Зменшення секреції альдостерону Decreased secretion of aldosterone

Зменшення реабсорбції глюкози Decreased glucose reabsorption

Глюкозурія Glucosuria

41 / 200
Після руйнування структури ЦНС тварина втратила орієнтувальні рефлекси. Що саме зруйновано? After the destruction of the CNS structure, the animal lost orientation reflexes. What exactly was destroyed?

Медiальнi ретикулярні ядра Medial reticular nuclei

Чотиригорбикова структура Four hump structure

Чорна речовина Black Matter

Латеральні вестибулярні ядра Lateral vestibular nuclei

Червоні ядра Red kernels

42 / 200
При визначенні основного обміну з’ясовано, що його величина у досліджуваного перевищує належну величину на 8%. Це означає, що інтенсивність процесів енергетичного метаболізму у досліджуваного: When determining the main metabolism, it was found that its value in the subject exceeds the appropriate value by 8%. This means that the intensity of energy metabolism processes in the subject:

Помірно підвищена Moderately increased

Помірно пригнічена Moderately depressed

Нормальна Normal

Суттєво пригнічена Significantly depressed

Суттєво підвищена Significantly increased

43 / 200
На судово-медичну експертизу надійшла кров дитини та передбачуваного батька для встановлення батьківства. Ідентифікацію яких хімічних компонентів необхідно здійснити в досліджуваній крові? The blood of the child and the alleged father was submitted for forensic examination to establish paternity. What chemical components must be identified in the examined blood?

ДНК DNA

мя-РНК mya-RNA

т-РНК t-RNA

м-РНК m-RNA

р-РНК p-RNA

44 / 200
Хворий 13-ти років скаржиться на загальну слабкість, запаморочення, втомлюваність. Спостерігається відставання у розумовому розвитку. При обстеженні виявлено високу концентрацію валіну, ізолейцину, лейцину в крові та сечі. Сеча специфічного запаху. Що може бути причиною такого стану? A 13-year-old patient complains of general weakness, dizziness, fatigue. There is a delay in mental development. The examination revealed a high concentration of valine, isoleucine, leucine in the blood and urine. Urine with a specific smell. What could be the cause of this condition?

Базедова хвороба Bazed disease

Хвороба кленового сиропу Maple syrup disease

Тирозиноз Tyrosinosis

Гістидинемія Histidinemia

Хвороба Аддісона Addison's disease

45 / 200
Пацієнт звернувся зі скаргами на напади утрудненого дихання, запаморочення. Працює на хімічному підприємстві з виробництва синильної кислоти. З порушенням функції якого ферменту можуть бути пов’язані вказані симптоми? The patient complained of attacks of difficulty breathing, dizziness. He works at a chemical plant producing hydrocyanic acid. The symptoms may be associated with a violation of the function of which enzyme?'

Цитохромоксидаза Cytochrome oxidase

Каталаза Catalase

Сукцинатдегідрогеназа Succinate dehydrogenase

Піруватдегідрогеназа Pyruvate dehydrogenase

Лактатдегідрогеназа Lactate dehydrogenase

46 / 200
У хворого в крові збільшена концентрація пірувату. Значна кількість його екскретується з сечею. Який авітаміноз спостерігається у хворого? The patient has an increased concentration of pyruvate in the blood. A significant amount of it is excreted in the urine. What type of vitamin deficiency is observed in the patient?

B1 B1

E E

B2 B2

B6 B6

B3 B3

47 / 200
Людина в стані спокою штучно примушує себе дихати часто і глибоко впродовж 3-4 хвилин. Як це відбивається на кислотно-лужній рівновазі організму? A person at rest artificially forces himself to breathe often and deeply for 3-4 minutes. How does this affect the acid-alkaline balance of the body?

Виникає дихальний ацидоз Respiratory acidosis occurs

Кислотно-лужна рівновага не змінюється The acid-base balance does not change

Виникає дихальний алкалоз Respiratory alkalosis occurs

Виникає метаболічний ацидоз Metabolic acidosis occurs

Виникає метаболічний алкалоз Metabolic alkalosis occurs

48 / 200
Пацієнтка з високим ступенем ожиріння у якості харчової добавки рекомендований карнітин для поліпшення 'спалювання' жиру. Яку безпосередню участь бере карнітин у процесі окислення жирів? A patient with a high degree of obesity is recommended carnitine as a dietary supplement to improve fat 'burning'. What is the direct role of carnitine in the process of fat oxidation?

Активація внутрішньоклітинного ліполізу Activation of intracellular lipolysis

Бере участь в одній з реакцій бета-окислення жирних кислот Participates in one of the reactions of beta-oxidation of fatty acids

Активація жирних кислот Activation of fatty acids

Транспорт жирних кислот з жирових депо до тканин Transport of fatty acids from fat depots to tissues

Транспорт жирних кислот з цитозоля до мітохондрій Transport of fatty acids from cytosol to mitochondria

49 / 200
В сечі новонародженого визначається цитрулін та високий рівень аміаку. Утворення якої речовини найімовірніше порушене у цього малюка? In the urine of a newborn, citrulline and a high level of ammonia are determined. The formation of which substance is most likely disturbed in this baby?

Сечовина Urea

Сечова кислота Uric acid

Креатинін Creatinine

Аміак Ammonia

Креатин Creatine

50 / 200
Причиною захворювання на пелагру може бути переважне харчування кукурудзою та зниження у раціоні продуктів тваринного походження. Відсутність у раціоні якої амінокислоти призводить до даної патології? The cause of pellagra disease can be a predominant diet of corn and a decrease in the diet of products of animal origin. The lack of which amino acid in the diet leads to this pathology?

Триптофан Tryptophan

Фенілаланін Phenylalanine

Ізолейцин Isoleucine

Метіонін Methionine

Гістидин Histidine

51 / 200
При лікуванні пародонтиту використовують препарати кальцію та гормон, що має здатність стимулювати мiнералiзацiю зу6ів та гальмувати резорбцію кісткової тканини, а саме: In the treatment of periodontitis, calcium preparations and a hormone are used, which has the ability to stimulate tooth mineralization and inhibit bone tissue resorption, namely:

Тироксин Thyroxine

Паратгормон Parathyroid hormone

Кальцитонін Calcitonin

Альдостерон Aldosterone

Адреналін Adrenaline

52 / 200
У пілота на висоті 14000 м трапилася аварійна розгерметизація кабіни. Який із видів емболії у нього розвинувся? The pilot had an emergency cabin depressurization at an altitude of 14,000 m. What type of embolism did he develop?

Газова Gas

Емболія стороннім тілом Foreign body embolism

Жирова Fat

Повітряна Air

Тромбоемболія Thromboembolism

53 / 200
Після введення місцевого анестетика у пацієнта розвинувся анафілактичний шок. Який механізм порушення кровообігу є провідним при цьому? After administration of a local anesthetic, the patient developed an anaphylactic shock. What mechanism of circulatory disturbance is the leading one?

Біль Pain

Гіперволемія Hypervolemia

Зниження скоротливої функції серця Decreased contractile function of the heart

Зменшення тонусу судин Decreased vascular tone

Активація симпато-адреналової системи Activation of sympatho-adrenal system

54 / 200
У підлітка було видалено зуб із застосуванням новокаїну. Через 10 хвилин у нього з’явилися блідість шкірних покривів, задишка, гіпотензія. При розвитку цієї реакції алерген на тканинних базофілах реагує з: A teenager had a tooth removed using novocaine. After 10 minutes, he developed pallor of the skin, shortness of breath, hypotension. During the development of this reaction, the allergen on tissue basophils responds with:

IgE IgE

IgA IgA

IgM IgM

Т-лімфоцитами by T-lymphocytes

IgD IgD

55 / 200
Чоловік 50-ти років хворіє на хронічний бронхіт, скаржиться на задишку при фізичному навантаженні, постійний кашель з відходженням харкотиння. При обстеженні діагностовано ускладнення - емфізема легень. Чим вона обумовлена? A 50-year-old man suffers from chronic bronchitis, complains of shortness of breath during physical exertion, a constant cough with expectoration. During the examination, a complication was diagnosed - emphysema of the lungs. What is it conditioned?

Зменшенням перфузії легень Reduced lung perfusion

Зниженням еластичних властивостей легень Decreasing the elastic properties of the lungs

Зменшенням розтяжності легень Reduced lung distensibility

Зменшенням альвеолярної вентиляції Reduced alveolar ventilation

Порушенням вентиляційно-перфузійного співвідношення в легенях Disruption of the ventilation-perfusion ratio in the lungs

56 / 200
Хвора надійшла до інфекційного відділення зі скаргами на нестримне блювання. Які порушення водно-сольового обміну будуть у хворої? The patient came to the infectious department with complaints of uncontrollable vomiting. What violations of water-salt metabolism will the patient have?

Гіперосмолярна дегідратація Hyperosmolar dehydration

Ізоосмолярна дегідратація Isoosmolar dehydration

Гіпоосмолярна дегідратація Hyposmolar dehydration

Гіпоосмолярна гіпергідратація Hyposmolar hyperhydration

Гіперосмолярна гіпергідратація Hyperosmolar hyperhydration

57 / 200
Робітник хімічної промисловості звернувся до лікаря зі скаргою на стирання емалі. Об’єктивно: розповсюджене руйнування коронок зубів з утворенням замісного дентину. Який діагноз найбільш вірогідний? A chemical industry worker went to the doctor with a complaint about enamel wear. Objectively: widespread destruction of tooth crowns with the formation of replacement dentine. What diagnosis is most likely?

Некроз твердих тканин зубів Necrosis of hard tissues of teeth

Клиноподібні дефекти Wedge defects

Середній карієс Average caries

Флюороз Fluorosis

Ерозія зубів Erosion of teeth

58 / 200
У чоловіка 30-ти років на нижній щелепі зліва в ділянці моляра визначається пухлиноподібне утворення щільної консистенції, дрібногорбисте, що значно деформує щелепу. Утворення місцями нечітко відшароване від кісткової тканини. Мікроскопічно: у стромі біоптату виявляються тяжі, фолікули, на периферії яких - клітини одонтогенного циліндричного епітелію, у центрі - зірчасті клітини, які нагадують пульпу емалевого органа. Який найбільш імовірний діагноз? A 30-year-old man has a tumor-like formation of a dense consistency, a small hump, which significantly deforms the jaw, on the lower jaw on the left in the area of the molar. The formation is vaguely detached from the bone tissue in places . Microscopically: in the stroma of the biopsy there are strands, follicles, on the periphery of which there are cells of the odontogenic cylindrical epithelium, in the center - stellate cells that resemble the pulp of the enamel organ. What is the most likely diagnosis?

Амелобластома Ameloblastoma

Остеобластокластома Osteoblastoclastoma

Аденоматоїдна пухлина Adenomatoid tumor

Первинний внутрішньокістковий рак Primary intraosseous cancer

Аденокарцинома Adenocarcinoma

59 / 200
У хворої, що страждала на вторинний сифіліс, з’явилися вогнища депігментації шкіри у верхніх відділах спини. Назвіть патологічний процес у шкірі: A patient suffering from secondary syphilis had foci of skin depigmentation in the upper back. Name the pathological process in the skin:

Лейкодерма Leukoderma

Лейкоплакія Leukoplakia

Дисплазія Dysplasia

Паракератоз Parakeratosis

Метаплазія Metaplasia

60 / 200
У хворого 14-ти років діагностована тріада Гетчинсона: зуби діжкоподібної форми, паренхіматозний кератит та глухота. Для якої хвороби характерні виявлені зміни? A 14-year-old patient was diagnosed with Hutchinson's triad: tub-shaped teeth, parenchymal keratitis and deafness. Which disease is characterized by the detected changes?

Опісторхоз Opistorchosis

Токсоплазмоз Toxoplasmosis

Проказа Leprosy

Сифіліс Syphilis

Туберкульоз Tuberculosis

61 / 200
Під час розтину тіла чоловіка 70-ти років, який помер від серцевої недостатності, виявлені деформовані, звужені коронарні артерії. На розрізі внутрішня поверхня артерій горбиста, стінка білувата, ламка, кам’янистої щільності. Про яку стадію атеросклерозу йдеться? During the autopsy of the body of a 70-year-old man who died of heart failure, deformed, narrowed coronary arteries were found. On the section, the inner surface of the arteries is bumpy, the wall is whitish, brittle, stony density. What stage of atherosclerosis are we talking about?

Атероматоз Atheromatosis

Ліпосклероз Liposclerosis

Виразкування Ultration

Ліпоїдоз Lipoidosis

Атерокальциноз Atherocalcinosis

62 / 200
На розтині тіла чоловіка похилого віку, який протягом останніх 2-х тижнів страждав від гострого розладу кишечнику, виявлені зміни у прямій та сигмоподібній кишках: на поверхні слизової оболонки відмічається коричнево-зелена плівка. Стінка кишки потовщена, порожнина різко звужена. Мікроскопічно виявляється проникаючий на різну глибину некроз слизової оболонки, некротичні маси пронизані нитками фібрину, з лейкоцитарною інфільтрацією. Який найбільш вірогідний діагноз? At the autopsy of the body of an elderly man who suffered from an acute intestinal disorder for the past 2 weeks, changes were found in the rectum and sigmoid colon: on the surface of the mucous membrane, brown-green film. The wall of the intestine is thickened, the cavity is sharply narrowed. Microscopically, necrosis of the mucous membrane penetrating to different depths is revealed, necrotic masses are penetrated by fibrin threads, with leukocyte infiltration. What is the most likely diagnosis?

Фібринозний коліт Fibrinous colitis

Фолікулярний коліт Follicular colitis

- -

Катаральний коліт Catarrhal colitis

Виразковий коліт Ulcerative colitis

63 / 200
На розтині тіла чоловіка, який помер на 5-у добу захворювання черевним тифом, виявлені наступні зміни: групові фолікули клубової кишки збільшені, повнокровні і виступають над слизовою оболонкою, на їх поверхні видно борозни та звивини. Гістологічно: повнокров’я і набряк тканини, наявність гранульом, які складаються з великих клітин зі світлою цитоплазмою і містять черевнотифозні палички. Про який період місцевих змін при черевному тифі можна думати? The autopsy of a man who died on the 5th day of typhoid fever revealed the following changes: the group follicles of the ileum are enlarged, full of blood and protruding above the mucous membrane, furrows and convolutions are visible on their surface. Histologically: full blood and swelling of the tissue, the presence of granulomas, which consist of large cells with light cytoplasm and contain typhoid bacilli. What period of local changes in typhoid fever can we think about?

Стадія чистих виразок Stage of pure ulcers

Стадія загоєння виразок Stage of ulcer healing

Стадія мозкоподібного набухання Stage of cerebral swelling

Стадія утворення виразок Stage of formation of ulcers

Стадія некрозу Stage of necrosis

64 / 200
У померлого від хронічної серцевої недостатності у віці 68-ми років на розтині виявлені деформовані, потовщені стулки мітрального клапану, що зрослися між собою, по краю змикання клапанів визначаються дрібні (1-2 мм) тромби. Який ендокардит був причиною розвитку хронічної серцевої недостатності? In a 68-year-old deceased from chronic heart failure, the autopsy revealed deformed, thickened leaflets of the mitral valve that had fused together, small (1-2 mm) thrombi. What kind of endocarditis caused the development of chronic heart failure?

Фібропластичний Fibroplastic

Поліпозно-виразковий Polypo-ulcerative

Дифузний Diffuse

Зворотньо-бородавчастий Reverse Warty

Гострий бородавчастий Sharp warty

65 / 200
У дитини 5-ти років підвищилася температура до 40o C, виник різкий головний біль, блювання, неспокій, озноб. Через 4 дні з’явився геморагічний висип на шкірі, олігурія та надниркова недостатність, що і стало причиною смерті. Під час бактеріологічного дослідження мазків з глотки виявлений менінгокок. Яка форма менінгококової інфекції була в хворого? A 5-year-old child's temperature rose to 40o C, a sharp headache, vomiting, restlessness, chills appeared. After 4 days, a hemorrhagic rash appeared on the skin , oliguria and adrenal insufficiency, which was the cause of death. Meningococcus was found during bacteriological examination of swabs from the throat. What form of meningococcal infection did the patient have?

Менінгоенцефаліт Meningoencephalitis

- -

Менінгококовий назофарингіт Meningococcal nasopharyngitis

Менінгококцемія Meningococcemia

Менінгококовий менінгіт Meningococcal meningitis

66 / 200
На розтині тіла чоловіка, який помер від гострої постгеморагічної анемії внаслідок легеневої кровотечі, було знайдено: макроскопічно - верхівки легень деформовані, на розрізі містять множинні білувато-сірі осередки діаметром 10-15 мм та множинні патологічні порожнини діаметром до 15 мм зі щільними стінками; мікроскопічно - в стінках порожнин розростання сполучної тканини з наявністю інфільтрату, який складається з епітеліоїдних клітин, багатоядерних гігантів та лімфоцитів. Який найбільш імовірний діагноз? On the autopsy of the body of a man who died of acute posthemorrhagic anemia as a result of pulmonary bleeding, it was found: macroscopically - the tops of the lungs are deformed, on section they contain multiple whitish-gray cells with a diameter 10-15 mm and multiple pathological cavities with a diameter of up to 15 mm with dense walls; microscopically, in the walls of the cavities, the growth of connective tissue with the presence of an infiltrate consisting of epithelioid cells, multinucleated giants, and lymphocytes. What is the most likely diagnosis?

Прогресуючий первинний туберкульозний комплекс Progressive primary tuberculosis complex

Гематогенний міліарний туберкульоз легень Hematogenous miliary pulmonary tuberculosis

Вторинний фіброзно-кавернозний туберкульоз Secondary fibro-cavernous tuberculosis

Гематогенно-дисемінований туберкульоз легень Hematogenously disseminated pulmonary tuberculosis

Первинний туберкульоз без ознак прогресування Primary tuberculosis without signs of progression

67 / 200
Для вакцинації використовують токсин, знешкоджений формальдегідом (0,4% розчин) при 37 — 40oC протягом чотирьох тижнів. Вперше такий препарат застосував для профілактики ди- фтерії Рамон. Що це за препарат? For vaccination, a toxin neutralized with formaldehyde (0.4% solution) is used at 37 — 40oC for four weeks. For the first time, such a drug was used to prevent diphtheria by Ramon. What kind of drug is this?

Вбита вакцина Killed vaccine

Ад’ювант Adjuvant

Антитоксична сироватка Antitoxic Serum

Анатоксин Anatoxin

Імуноглобулін Immunoglobulin

68 / 200
Пацієнтка 65-ти років з хронічною серцевою недостатністю тривалий час лікується дигітоксином, самостійно визначаючи дозу. Надійшла до стаціонару із скаргами на погіршення загального стану, появу аритмії, нудоту, зменшення виділення сечі, безсоння. З чого слід розпочати лікування хворої? A 65-year-old patient with chronic heart failure has been treated with digitoxin for a long time, independently determining the dose. She came to the hospital with complaints of deterioration of her general condition, appearance of arrhythmia, nausea, decrease in urine output, insomnia. Where should the treatment of the patient begin?

Призначити дигоксин Prescribe digoxin

Зменшити дозу дигітоксину Reduce the dose of digitoxin

Ввести розчин кальцію глюконату внутрішньовенно Enter calcium gluconate solution intravenously

Відмінити дигітоксин Cancel Digitoxin

Призначити строфантин внутрішньовенно Prescribe strophantin intravenously

69 / 200
В хірургічному відділенні стоматологічної поліклініки готують хворого до екстракції зуба. Який препарат слід додати до розчину місцевого анестетика для подовження його дії? In the surgical department of the dental polyclinic, a patient is being prepared for tooth extraction. What drug should be added to the local anesthetic solution to prolong its effect?

Адреналіну гідрохлорид Adrenaline hydrochloride

Сальбутамол Salbutamol

Октадин Octadine

Норадреналіну гідротартрат Noradrenaline hydrotartrate

Ізадрин Izadrin

70 / 200
Хворий надійшов до хірургічного відділення з діагнозом гострий панкреатит. Розпочато консервативне лікування. Призначення якого препарату є патогенетично обґрунтованим? The patient was admitted to the surgical department with a diagnosis of acute pancreatitis. Conservative treatment was started. The prescription of which drug is pathogenetically justified?

Контрикал Contrical

Хімотрипсин Chymotrypsin

Трипсин Trypsin

Фібринолізин Fibrinolysin

Панкреатин Pancreatin

71 / 200
Хвора скаржиться на біль у голені, який підсилюється під час ходи. Об’єктивно: відмічається набряк та почервоніння по ходу вени. Лікар призначив антикоагулянт прямої дії для місцевого вживання. Який препарат можна використати з цією метою? The patient complains of pain in the shin, which intensifies while walking. Objectively: swelling and redness along the course of the vein are noted. The doctor prescribed a direct-acting anticoagulant for local use . What drug can be used for this purpose?

Мазь троксевазинова Troxevasin ointment

Мазь гепаринова Heparin ointment

Мазь саліцилова Salicylic ointment

Тромбін Thrombin

Мазь бутадіонова Butadione ointment

72 / 200
До поліклініки звернувся хворий зі скаргами на біль за грудниною, зади-тттку і серцебиття. Після обстеження лікар діагностував у хворого ІХС і призначив верапаміл. Який механізм дії даного препарату? A patient came to the polyclinic with complaints of chest pain, back pain, and heart palpitations. After the examination, the doctor diagnosed the patient with coronary artery disease and prescribed verapamil. What is the mechanism of action of this drug ?

Блокує калієві канали Blocks potassium channels

Блокує в-адренорецептори Blocks v-adrenoceptors

Блокує а-адренорецептори Blocks α-adrenoceptors

Блокує кальцієві канали Blocks calcium channels

Блокує натрієві канали Blocks sodium channels

73 / 200
До стоматолога звернувся пацієнт з артритом скронево-нижньощелепного суглоба. Лікарстоматолог призначив мазь із диклофенак-натрієм. Який механізм дії цього лікарського засобу? A patient with arthritis of the temporomandibular joint came to the dentist. The dentist prescribed an ointment with diclofenac sodium. What is the mechanism of action of this medicine?

Активація циклооксигенази Activation of cyclooxygenase

Пригнічення циклооксигенази Inhibition of cyclooxygenase

Блокада опіатних рецепторів Opiate receptor blockade

Пригнічення фосфоліпази Inhibition of phospholipase

Активація опіатних рецепторів Activation of opiate receptors

74 / 200
Новонароджений не зробив перший вдих. При патологоанатомічному розтині тіла встановлено, що при вільних дихальних шляхах легені не розправилися. Що з наведеного могло бути причиною цього? The newborn did not take his first breath. At the post-mortem examination of the body, it was established that the lungs did not expand with free airways. Which of the following could be the reason for this?

Збільшення розміру альвеол Increase in alveolar size

Розрив бронхів Rupture of bronchi

Звуження бронхів Bronchi narrowing

Потовщення плеври Pleural thickening

Відсутність сурфактанту Lack of surfactant

75 / 200
У хворого спостерігається збільшення проникності стінок кровоносних судин із розвитком підвищеної кровоточивості ясен, виникнення дрібнокра-плинних крововиливів на шкірі, випадіння зубів. Яка патологія має місце у хворого? The patient has an increase in the permeability of the walls of blood vessels with the development of increased gum bleeding, the appearance of small-flowing hemorrhages on the skin, tooth loss. What pathology does the patient have?

Гіпервітаміноз D Hypervitaminosis D

Гіповітаміноз C Hypovitaminosis C

Гіпервітаміноз C Hypervitaminosis C

Гіповітаміноз D Hypovitaminosis D

Гіповітаміноз A Hypovitaminosis A

76 / 200
У хворого внаслідок травми ушкоджені передні корінці спинного мозку. Які структури постраждають при цьому? The patient's front roots of the spinal cord are damaged as a result of the injury. What structures will be affected?

Центральні відростки чутливих нейронів спинномозкових вузлів Central processes of sensory neurons of spinal nodes

Аксони мотонейронів і аксони нейронів бокових рогів Axons of motor neurons and axons of lateral horn neurons

Аксони нейронів бокових рогів Axons of lateral horn neurons

Дендрити нейронів спинномозкових вузлів Dendrites of neurons of spinal nodes

Периферійні відростки чутливих нейронів спинномозкових вузлів Peripheral processes of sensitive neurons of spinal nodes

77 / 200
В стоматологічному кабінеті у хворого розвинувся напад бронхіальної астми. Лікар застосував препарат з групи ^-адреноміметиків у вигляді інгаляцій. Який препарат був застосований? In the dental office, the patient developed an attack of bronchial asthma. The doctor used a drug from the group of ^-adrenomimetics in the form of inhalations. What drug was used?

Атропіну сульфат Atropine sulfate

Ефедрину гідрохлорид Ephedrine hydrochloride

Еуфілін Euphilin

Сальбутамол Salbutamol

Адреналіну гідрохлорид Adrenaline hydrochloride

78 / 200
Хворий скаржиться на головний біль, утруднене дихання. Рентген-дослідження підтвердило діагноз -фронтит (запалення лобової пазухи). В якому носовому ході при огляді порожнини носа можуть спостерігатись гнійні виділення? The patient complains of a headache, difficulty breathing. An X-ray study confirmed the diagnosis of frontitis (inflammation of the frontal sinus). In which nasal passage, when examining the nasal cavity, purulent masses can be observed selection?

Верхній Upper

Нижній Lower

Над верхньою носовою раковиною Above superior turbinate

Середній Average

Загальний General

79 / 200
Під час дослідження мазку крові, взятого у хворого і забарвленого за Ро-мановським, лікар виявив найпростіші і встановив діагноз - хвороба Круза-Чагаса. Яке найпростіше викликало захворювання у даного хворого? During the study of a blood smear taken from a patient and stained according to Romanovsky, the doctor found the simplest protozoan and diagnosed it as Cruz-Chagas disease. Which protozoan caused the disease in this patient?

Leishmania tropica Leishmania tropica

Toxoplasma gondii Toxoplasma gondii

Trypanosoma brucei Trypanosoma brucei

Leishmania donovani Leishmania donovani

Trypanosoma cruzi Trypanosoma cruzi

80 / 200
У повітрі приміщення збільшений вміст вуглекислого газу. Як зміниться дихання (глибина і частота) у людини, що увійшла в це приміщення? The air in the room has increased carbon dioxide content. How will the breathing (depth and frequency) of a person who entered this room change?

Збільшаться глибина і частота Depth and frequency will increase

Зменшиться глибина Decrease depth

Зменшиться частота Frequency will decrease

Збільшиться частота Frequency will increase

Збільшиться глибина The depth will increase

81 / 200
У хворої 38-ми років після прийому аспірину і сульфаніламідів спостерігається посилений гемоліз еритро- цитів, який викликаний недостатністю глюкозо-6-фосфатдегідрогенази. Порушенням утворення якого коферменту зумовлена ця патологія? In a 38-year-old patient, after taking aspirin and sulfonamides, increased hemolysis of erythrocytes is observed, which is caused by a deficiency of glucose-6-phosphate dehydrogenase. Violation of the formation of which coenzyme is caused by this pathology?

фад-н2 fad-n2

У6іхінон U6iquinone

ФМН-Н2 ФМН-Н2

Шридоксальфосфат Sridoxal phosphate

НАДФ-Н NADF-H

82 / 200
При підозрі на туберкульоз хворій дитині зробили пробу Манту Через 24 години у місці введення алергену з’явилися припухлість, гіперемія і болісність. Які основні компоненти визначають розвиток цієї реакції організму? On suspicion of tuberculosis, a sick child was given a Mantoux test. After 24 hours, swelling, hyperemia, and pain appeared at the site of allergen injection. What are the main components that determine the development of this reaction of the body ?

Плазматичні клітини, Т-лімфоцити і лімфокіни Plasma cells, T-lymphocytes and lymphokines

Гранулоцити, Т-лімфоцити і IgG Granulocytes, T-lymphocytes and IgG

Макрофаги, В-лімфоцити і моноцити Macrophages, B-lymphocytes and monocytes

В-лімфоцити, IgM B-lymphocytes, IgM

Мононуклеари, Т-лімфоцити і лім-фокіни Mononuclear cells, T-lymphocytes and lymphokines

83 / 200
У дітей часто можна спостерігати утруднене носове дихання, яке пов’язане з надмірним розвитком лімфої-дної тканини слизової оболонки глотки. Розростання яких мигдаликів може спричинити це явище? Difficult nasal breathing can often be observed in children, which is associated with excessive development of the lymphoid tissue of the mucous membrane of the pharynx. The growth of which tonsils can cause this phenomenon?

Tonsilla lingualis Tonsilla lingualis

Усіх названих мигдаликів All named tonsils

Tonsilla pharyngea Tonsilla pharyngea

Tonsilla palatina Tonsilla palatina

Tonsilla tubaria Tonsilla tubaria

84 / 200
У крові хворого загальна кількість лейкоцитів 90 • 109/л. В лейкоцитарній формулі: е.- 0%, б.- 0%, ю.- 0%, п.- 2%, с.- 20%, лімфобласти - 1%, пролімфо-цити - 2%, лімфоцити - 70%, м.- 5%, наявні клітини Боткіна-Іумпрехта. Клінічно: збільшені шийні, підщелепні лімфатичні вузли. Для якої патології характерна така клінічна картина? The total number of leukocytes in the patient's blood is 90 • 109/l. In the leukocyte formula: e.- 0%, b.- 0%, yu.- 0% , p.- 2%, p.- 20%, lymphoblasts - 1%, prolymphocytes - 2%, lymphocytes - 70%, m. - 5%, Botkin-Iumprecht cells are present. Clinically: enlarged cervical, submandibular lymph nodes . For what pathology is this clinical picture typical?

Інфекційний мононуклеоз Infectious mononucleosis

Лімфогранульоматоз Lymphogranulomatosis

Гострий лімфолейкоз Acute lymphocytic leukemia

Хронічний мієлолейкоз Chronic myelogenous leukemia

Хронічний лімфолейкоз Chronic lymphocytic leukemia

85 / 200
У деяких клінічно здорових людей в умовах високогір’я виявляються ознаки анемії. У крові в них виявляють серпоподібні еритроцити. Іенотип цих людей: Some clinically healthy people in the highlands show signs of anemia. Sickle-shaped erythrocytes are found in their blood. Ienotype of these people:

аа aa

ХсХс XxXx

ХСХс ХСХс

Аа Ah

АА AA

86 / 200
Хвора на хронічний гепатит скаржиться на підвищення чутливості до барбiтyратiв, які раніше переносила без симптомів інтоксикації. З порушенням якої функції пєчінки це пов’язане у найбільшій мірі? A patient with chronic hepatitis complains of increased sensitivity to barbiturates, which she previously tolerated without symptoms of intoxication. Which liver function disorder is this most likely associated with?

Іемопоетична Hematopoietic

Іемодинамічна Iemodynamic

Метаболічна Metabolic

Утворення жовчі Bile Formation

Фагоцитарна Phagocytic

87 / 200
Бактеріологічне дослідження гнійних виділень з уретри з’ясувало присутність бактерій, які за Грамом фарбувалися негативно, нагадували кавові зернини, розкладали глюкозу і мальтозу до кислоти. Розташовувалися в лейкоцитах. До збудників якої хвороби їх віднести? Bacteriological examination of purulent secretions from the urethra revealed the presence of bacteria that stained negatively on Gram, resembled coffee beans, decomposed glucose and maltose into acid. They were located in leukocytes. To the causative agents of which disease should they be attributed?

Сифіліс Syphilis

Меліоїдоз Melioidosis

Іонорея Ionorrhea

Венеричний лімфогранулематоз Venereal lymphogranulomatosis

М ’який шанкр Mild chancre

88 / 200
При обстеженні хворого 35-ти років проведено гістологічне дослідження пунктату червоного кісткового мозку і виявлено значне зменшення кількості мегакаріоцитів. До яких змін периферичної крові це призведе? During the examination of a 35-year-old patient, a histological examination of the red bone marrow punctate was performed and a significant decrease in the number of megakaryocytes was found. What changes in the peripheral blood will this lead to?

Агранулоцитоз Agranulocytosis

Лейкопенія Leukopenia

Лейкоцитоз Leukocytosis

Тромбоцитоз Thrombocytosis

Тромбоцитопенія Thrombocytopenia

89 / 200
Згідно правила сталості числа хромосом, кожний вид більшості тварин має певне і стале число хромосом. Механізмом, що підтримує цю сталість при статевому розмноженні організмів, є: According to the rule of constancy of the number of chromosomes, each species of most animals has a certain and constant number of chromosomes. The mechanism that supports this constancy during the sexual reproduction of organisms is:

Мейоз Meiosis

Амітоз Amitosis

Шизогонія Schizogony

Брунькування Budding

Регенерація Regeneration

90 / 200
Під час барбітурового наркозу у хворого 65-ти років почало прогресувати пригнічення дихання. Анестезіолог зробив внутрішньовенну ін’єкцію 10 мл 0,5% розчину бемегріду. Стан хворого покращився, об’єм легеневої вентиляції збільшився. Яке явище полягає в основі взаємодії цих препаратів? During barbiturate anesthesia in a 65-year-old patient, respiratory depression began to progress. The anesthesiologist administered an intravenous injection of 10 ml of 0.5% bemegrid solution. The patient's condition improved , the volume of pulmonary ventilation increased. What phenomenon is the basis of the interaction of these drugs?

Антагонізм однобічний Antagonism is unilateral

Антагонізм прямий Antagonism is direct

Синергізм прямий Synergism is direct

Антагонізм непрямий Antagonism is indirect

Синергізм непрямий Synergism is indirect

91 / 200
Під час проведення трахеотомії у хворого 45-ти років, який потрапив до реанімаційного відділення лікарні з набряком гортані, було випадково перерізано яремну венозну дугу, яка лежить у: During a tracheotomy in a 45-year-old patient who was admitted to the hospital's intensive care unit with laryngeal edema, the jugular venous arch was accidentally cut, which lies in:

Spatium interscalenum Spatium interscalenum

Spatium antescalenum Spatium antescalenum

Spatium retropharyngeale Spatium retropharyngeale

Spatium interaponeuroticum suprasternale Spatium interaponeuroticum suprasternale

Spatium pretracheale Spatium pretracheale

92 / 200
Під час виконання фізичного навантаження людина менш чутлива до болю. Причиною цього є активація: When performing physical activity, a person is less sensitive to pain. The reason for this is activation:

Функції щитоподібної залози Thyroid functions

Антиноцицептивної системи Antinociceptive system

Функції наднирників Adrenal functions

Симпато-адреналової системи Sympatho-adrenal system

Ноцицептивної системи Nociceptive system

93 / 200
У чоловіка 35-ти років через 30 хвилин після автомобільної аварії виявлена масивна травма нижніх кінцівок без значної зовнішньої кровотечі. Постра-ждалий знаходиться у збудженому стані. Який компонент патогенезу травматичного шоку є у пацієнта провідним і вимагає негайного корегування? 30 minutes after a car accident, a 35-year-old man was found to have a massive injury to the lower extremities without significant external bleeding. The victim is in an excited state. What component of pathogenesis traumatic shock is the patient's leading and requires immediate correction?

Внутрішня плазмовтрата Internal plasma loss

Порушення функції органів Organ dysfunction

Внутрішня кровотеча Internal bleeding

Інтоксикація Intoxication

Біль Pain

94 / 200
У хворого діагностовано себорейний дерматит, пов’язаний із дефіцитом вітаміну H (біотину). Порушення активності якого з перелічених ферментів спостерігається у хворого? The patient was diagnosed with seborrheic dermatitis associated with vitamin H (biotin) deficiency. Which of the listed enzymes is impaired in the patient?

Піруватдекарбоксилаза Pyruvate decarboxylase

Карбамоїлфосфатсинтетаза Carbamoyl phosphate synthetase

Амінотрансферази Aminotransferases

Алкогольдегідрогеназа Alcohol dehydrogenase

Ацетил-КоА-карбоксилаза Acetyl CoA carboxylase

95 / 200
Хворий 60-ти років звернувся до лікаря зі скаргами на загруднинний 6іль після фізичного навантаження. Лікар призначив нітрогліцерин. Після застосування препарату загруднинний біль зник, проте з’явився сильний біль голови. Який можливий механізм цієї побічної дії? A 60-year-old patient turned to the doctor with complaints of chest pain after physical exertion. The doctor prescribed nitroglycerin. After using the drug, the chest pain disappeared, but a strong headache. What is the possible mechanism of this side effect?

Блокада фосфодиестерази Phosphodiesterase blockade

Зменшення накопичення іонів кальцію Decreased accumulation of calcium ions

Підвищення внутрішньочерепного тиску Increased intracranial pressure

Блокада а-адренорецепторів A-adrenoceptor blockade

Гальмування утворення медіаторів у мозку Inhibition of the formation of mediators in the brain

96 / 200
При копрологічному дослідженні у фекаліях хворої знайдено яйця дрібних розмірів із кришечкою. З анамнезу відомо, що жінка часто вживає рибні страви. Який сисун паразитує в організмі? During the coprological examination, small-sized eggs with a lid were found in the patient's feces. From the anamnesis, it is known that the woman often eats fish dishes. What type of sisun is a parasite in the body?

Легеневий Pulmonary

Кров’яний Bloody

Котячий Feline

Ланцетоподібний Lancet

Печінковий Hepatic

97 / 200
Відпочиваючи на дачі, хлопчик знайшов павука з такими морфологічними особливостями: довжина - 2 см, округле черевце чорного кольору, на спинному боці якого видно червоні плямочки у два ряди, чотири пари членистих кінцівок вкриті дрібними чорними волосками. Визначте дане членистоноге: While resting in the country, the boy found a spider with the following morphological features: length - 2 cm, rounded black abdomen, on the back side of which red spots in two rows are visible, four pairs of articulated limbs are covered with small black hairs. Identify this arthropod:

Каракурт Karakurt

Фаланги Falangs

Кліщ Tick

Тарантул Tarantula

Скорпіон Scorpio

98 / 200
Під час клінічного обстеження пацієнтки виявлено зниження основного обміну на 40%, збільшення маси тіла, зниження температури тіла, одутлість обличчя, порушення статевих функцій, млявість і апатія, зниження інтелекту. Яке порушення функції і якої залози внутрішньої секреції призводить до появи даних симптомів? During the clinical examination of the patient, a decrease in basic metabolism by 40%, an increase in body weight, a decrease in body temperature, swelling of the face, sexual dysfunction, lethargy and apathy, decrease intelligence. What dysfunction and which gland of internal secretion leads to the appearance of these symptoms?

Гіперфункція гіпофізу Hyperfunction of the pituitary gland

Гіпофункція епіфізу Pineal hypofunction

Гіпофункція паращитоподібних залоз Hypofunction of parathyroid glands

Гіперфункція щитоподібної залози Hyperthyroidism

Гіпофункція щитоподібної залози Hypothyroidism

99 / 200
У пацієнта з пошкодженням промі- жного мозку виявлено порушення слуху. Які структури при цьому ушкоджені? A hearing impairment was detected in a patient with damage to the diencephalon. What structures are damaged?

Латеральні колінчасті тіла таламуса Lateral geniculate bodies of the thalamus

Медіальні колінчасті тіла таламуса Medial geniculate bodies of the thalamus

Передні ядра гіпоталамуса Anterior nuclei of the hypothalamus

Інтраламінарні ядра гіпоталамуса Intralaminar nuclei of the hypothalamus

Медіальні ядра гіпоталамуса Medial nuclei of the hypothalamus

100 / 200
У хворого внаслідок травми розвинувся травматичний шок. Хворий метушливий, багатослівний, блідий. АТ-140/90 мм рт.ст., Ps- 120/хв. Якій стадії шоку відповідає цей стан? The patient developed traumatic shock as a result of the injury. The patient is fussy, talkative, pale. BP-140/90 mm Hg, Ps- 120/min. What does this state correspond to the stage of shock?

Кінцева Final

Латентний період Latent period

Термінальна Terminal

Еректильна Erectile

Торпідна Torped

101 / 200
Хворому з токсичним паралічем дихального центру для його стимуляції вводили декілька разів кордіамін. Який побічний ефект може виникнути при цьому? A patient with toxic paralysis of the respiratory center was injected several times with cordiamine to stimulate it. What side effect can occur?

Тонічні судоми Tonic seizures

Бронхоспазм Bronchospasm

Клонічні судоми Clonic seizures

Аритмія Arrhythmia

Колапс Collapse

102 / 200
У хворого, який знаходиться на лікуванні з приводу вірусного гепатиту В, з’явилися ознаки печінкової недостатності. Які зміни крові, що свідчать про порушення білкового обміну, найбільш імовірно будуть спостерігатися у даному випадку? A patient who is being treated for viral hepatitis B has developed signs of liver failure. What changes in the blood that indicate a violation of protein metabolism are most likely will be observed in this case?

Абсолютна гіперглобулінемія Absolute hyperglobulinemia

Абсолютна гіпоальбумінемія Absolute hypoalbuminemia

Абсолютна гіперфібриногенемія Absolute hyperfibrinogenemia

Білковий склад крові не змінений The protein composition of the blood has not changed

Абсолютна гіперальбумінемія Absolute hyperalbuminemia

103 / 200
У чоловіка 30-ти років перед операцією визначили групу крові. Кров резус-позитивна. Реакція аглютинації еритроцитів не відбулася зі стандартними сироватками груп 0(І), А(ІІ), В(ІІІ). Досліджувана кров за системою АВ0 належить до групи: The blood group of a 30-year-old man was determined before the operation. The blood is Rh-positive. The erythrocyte agglutination reaction did not occur with standard sera of groups 0(I), A( ІІ), В(ІІІ). The studied blood according to the AB0 system belongs to the group:

- -

0(І) 0(And)

В(ІІІ) B(ІІІ)

А(ІІ) А(II)

АВ(ІУ) AB(IU)

104 / 200
Під час травми грудної клітки по- шкоджена хрящова частина ребра. За рахунок якого шару охрястя відбувається регенерація хряща? During a chest injury, the cartilage part of the rib was damaged. At the expense of which cartilage layer does cartilage regeneration occur?

Волокна Шарпея Sharpei fiber

Хондрогенний Chondrogenic

Колагеновий Collagen

Еластичний Elastic

Фіброзний Fibrous

105 / 200
Після парентерального введення препарату в пацієнта розвинувся коматозний стан, спостерiгається дихання типу Чейн-Стокса, зіниці різко звужену колінний рефлекс збережений. Який препарат міг спричинити отруєння? After parenteral administration of the drug, the patient developed a comatose state, Cheyne-Stokes breathing was observed, the pupils were sharply narrowed, the knee reflex was preserved. What drug could have caused the poisoning?

Морфіну гідрохлорид Morphine hydrochloride

Аміназин Aminazine

Фенобарбітал Phenobarbital

Сибазон Sibazone

Анальгін Analgin

106 / 200
На гістологічному препараті підщелепної залози видно вивідну протоку. Слизова оболонка протоки вистелена низьким кубічним епітелієм, клітини якого мають слабо розвинуті органели. Що це за вивідна протока? The excretory duct is visible on the histological preparation of the submandibular gland. The mucous membrane of the duct is lined with a low cuboidal epithelium, the cells of which have poorly developed organelles. What kind of excretory duct is this?

Міжчасточкова Interparticle

Посмугована Striped

- -

Вставна Insertive

Загальна вивідна Total output

107 / 200
Під час голодування нормальний рівень глюкози підтримується за рахунок активації глюконеогенезу. Назвіть речовину яка може використовуватись як субстрат для цього процесу: During fasting, the normal level of glucose is maintained due to the activation of gluconeogenesis. Name the substance that can be used as a substrate for this process:

Аланін Alanine

Сечовина Urea

Гуанін Guanin

Амоніак Ammonia

Аденін Adenine

108 / 200
У хворого після застосування крапель, що містять атропін, виникло стійке розширення зіниці. Роботу якого м’яза блоковано? After using drops containing atropine, the patient developed persistent dilation of the pupil. Which muscle is blocked?

Війковий Ciliary

Звужувач зіниці Pupil Constrictor

Прямий Direct

Розширювач зіниці Pupil Dilator

Косий Slanted

109 / 200
Жінка під час пологів втратила близько 800 мл крові. Відзначаються тахікардія, АТ100/70 мм рт.ст., тахі-пное до 28/хв. Який тип гіпоксії пер- винно розвивається при такій клінічній ситуації? During childbirth, a woman lost about 800 ml of blood. Tachycardia, blood pressure 100/70 mm Hg, tachypnea up to 28/min are noted. What type of hypoxia primarily develops in such a clinical situation?

Серцево-судинна Cardiovascular

Кров’яна Bloody

Змішана Mixed

Дихальна Respiratory

Тканинна Fabric

110 / 200
Хворому для лікування ІХС лікар призначив антиангінальний засіб, який активує гуанілатциклазу і накопичує цГМФ у клітинах міокарду. Який це засіб? For the treatment of coronary heart disease, the doctor prescribed an anti-anginal drug that activates guanylate cyclase and accumulates cGMP in myocardial cells. What is this drug?

Верапаміл Verapamil

Панангін Panangin

Дипіридамол Dipyridamole

Ізосорбіду мононітрат Isosorbide mononitrate

Валідол Validol

111 / 200
У хворого, що надійшов до інфекційного відділення зі скаргами на судомне скорочення м’язів обличчя, з садна правої нижньої кінцівки були виділені бактерії з термінальним розташуванням спор, що надає їм вигляд 'барабанних паличок'. Яким бактеріям притаманні дані властивості? In a patient who came to the infectious disease department with complaints of convulsive contraction of the facial muscles, bacteria with a terminal location of spores were isolated from the wound of the right lower limb, which gives they look like 'drumsticks'. Which bacteria have these properties?

Clostridium botulinum Clostridium botulinum

Bacillus cereus Bacillus cereus

Clostridium tetani Clostridium tetani

Bacillus anthracis Bacillus anthracis

Clostridium perfringens Clostridium perfringens

112 / 200
Хворий звернувся з приводу карбункула на обличчі. Об’єктивно: нещільний, без болю набряк підшкірної клітковини, у центрі карбункулу чорний струп, по периферії навколо карбункула везикулярні висипання. Бактеріологічне дослідження довело наявність нерухомих стрептобацил, які здатні утворювати капсули. Які мікроорганізми є збудниками даної хвороби? The patient applied for a carbuncle on the face. Objectively: loose, painless swelling of the subcutaneous tissue, a black scab in the center of the carbuncle, vesicular rashes on the periphery around the carbuncle. Bacteriological research proved the presence of immobile streptobacilli, which are capable of forming capsules. What microorganisms are the causative agents of this disease?

Bacillus subtilis Bacillus subtilis

Staphylococcus aureus Staphylococcus aureus

Bacillus antracis Bacillus anthracis

Bacillus anthracoides Bacillus anthracoides

Bacillus megaterium Bacillus megaterium

113 / 200
На схемі представлена екзокринна залоза, яка має нерозгалужену вивідну протоку, в яку відкривається один кінцевий відділ у вигляді одного мішечка. Як буде називатися така залоза відповідно до морфологічної класифікації екзокринних залоз? The diagram shows an exocrine gland that has an unbranched excretory duct into which one terminal section opens in the form of a single pouch. What will such a gland be called according to the morphological classification of exocrine glands?

Проста нерозгалужена альвеолярна Simple unbranched alveolar

Складна нерозгалужена альвеолярно-трубчаста Complex unbranched alveolar-tubular

Складна розгалужена альвеолярна Complex branched alveolar

Проста розгалужена трубчаста Simple Branched Tubular

Складна нерозгалужена альвеолярна Complex unbranched alveolar

114 / 200
У хворого з порушенням згортання кровї виявлений тромбоз однієї з гілок нижньої брижової артерії. Який відділ кишечнику вражений? Thrombosis of one of the branches of the lower mesenteric artery was detected in a patient with a blood coagulation disorder. Which part of the intestine is affected?

Colon ascendens Colon ascendens

Colon transversum Colon transversum

Ileum Ileum

Colon sigmoideum Colon sigmoideum

Caecum Caecum

115 / 200
Дослідженнями останніх дєсятиріч встановлено, що безпосередніми 'виконавцями'апоптозу в клітині є особливі ферменти - каспази. В утворенні одного з них бере участь цитохром С. Вкажіть його функцію в нормальній клітині: Research over the last ten years has established that the immediate 'executors' of apoptosis in the cell are special enzymes - caspases. Cytochrome C is involved in the formation of one of them. Specify its function in normal cell:

Компонент H+-АТФазної системи Component of the H+-ATPase system

Фермент дихального ланцюга переносу електронів Enzyme of the respiratory chain of electron transfer

Фермент бета-окислення жирних кислот Enzyme beta-oxidation of fatty acids

Фермент ЦТК CTC Enzyme

Компонент піруватдегідрогеназної системи Component of pyruvate dehydrogenase system

116 / 200
До хірургічного відділення госпіталізовано хворого для оперативного втручання. Йому необхідно провести нейролептанальгезію. Який з препаратів доцільно застосувати в комбінації з фентанілом для нейролептанальгезії? A patient was admitted to the surgical department for surgery. He needs neuroleptanalgesia. Which drug should be used in combination with fentanyl for neuroleptanalgesia?

Фраксипарин Fraxiparin

Пілокарпін Pilocarpine

Холосас Holosas

Дроперидол Droperidol

Сальбутамол Salbutamol

117 / 200
Хворому на ревматоїдний артрит для попередження можливого негативного впливу на слизову шлунка призначили препарат із групи нестероїдних протизапальних засобів - селективний інгібітор ЦОГ-2. Вкажіть препарат: A patient with rheumatoid arthritis was prescribed a drug from the group of nonsteroidal anti-inflammatory drugs - a selective COX-2 inhibitor to prevent a possible negative effect on the gastric mucosa. Specify the drug:

Целекоксиб Celecoxib

Ібупрофен Ibuprofen

Бутадіон Butadione

Ацетилсаліцилова кислота Acetylsalicylic acid

Анальгін Analgin

118 / 200
При лікуванні багатьох захворювань використовується фармацевти- чний препарат кокарбоксилаза (тіамін-пірофосфат) для забезпечення клітин енергією. Який метаболічний процес при цьому активується? In the treatment of many diseases, the pharmaceutical drug cocarboxylase (thiamine pyrophosphate) is used to provide cells with energy. What metabolic process is activated?

Декарбоксилювання амінокислот Decarboxylation of amino acids

Дезамінування глутамату Glutamate deamination

Декарбоксилювання біогенних амінів Decarboxylation of biogenic amines

Детоксикація шкідливих речовин у печінці Detoxification of harmful substances in the liver

Окисне декарбоксилювання пірувату Oxidative decarboxylation of pyruvate

119 / 200
Хворій, що хворіє на інсуліноне-залежний цукровий діабет, призначено всередину глібенкламід. Вкажіть механізм гіпоглікемічної дії цього препарату: A patient suffering from insulin-dependent diabetes is prescribed glibenclamide orally. Specify the mechanism of hypoglycemic action of this drug:

Посилює утилізацію глюкози периферичними тканинами Strengthens utilization of glucose by peripheral tissues

Пригнічує а-глюкозидазу і розпад полісахаридів Suppresses α-glucosidase and the breakdown of polysaccharides

Пригнічує глюконеогенез у печінці Suppresses gluconeogenesis in the liver

Стимулює виділення ендогенного інсуліну в-клітинами Stimulates release of endogenous insulin by b-cells

Пригнічує всмоктування глюкози у кишечнику Suppresses the absorption of glucose in the intestine

120 / 200
При дегельмінтизації у хворого виявлені довгі фрагменти гельмінта, що має членисту будову. Зрілі членики прямокутної форми 30х12 мм, матка закритого типу у вигляді стовбура, від якого відходять 17-35 бічних відгалужень. Визначте вид гельмінта: During deworming, long fragments of a helminth with an articulated structure were found in the patient. Mature segments of a rectangular shape 30x12 mm, a closed-type uterus in the form of a trunk, from which 17- 35 side branches. Identify the type of helminth:

Ціп ’як карликовий Zip 'like a dwarf'

Ехінокок Echinococcus

Альвеокок Alveococcus

Ціп’як неозброєний The hook is unarmed

Ціп ’як озброєний Cip 'as armed'

121 / 200
У хворого з виразковою хворобою після проведеного лікування нормалізувалися травлення, зник біль, поліпшився настрій. Однак через кілька тижнів знову з’явився біль у епігастрії, печія, відрижка кислим. Як охарактеризувати такий перебіг хвороби? After the treatment, digestion normalized in a patient with peptic ulcer, the pain disappeared, and the mood improved. However, after a few weeks, pain in the epigastrium, heartburn, and sour belching reappeared How to characterize this course of the disease?

Продромальний період Prodromal period

Ускладнення Complication

Період ремісії Period of remission

Латентний період Latent period

Рецидив Relapse

122 / 200
Під час гемотрасфузій рекомендується переливати кров лише відповідної групи. Належність до даної (за системою АВ0) групи зумовлюють: During hemotransfusions, it is recommended to transfuse blood only of the appropriate group. Belonging to this (according to the AB0 system) group is conditioned by:

Вуглеводні детермінанти мембран лейкоцитів Carbohydrate determinants of leukocyte membranes

Вуглеводні детермінанти мембран еритроцитів Carbohydrate determinants of erythrocyte membranes

Білки сироватки крові Blood serum proteins

Білково-полісахаридні компоненти лейкоцитів Protein-polysaccharide components of leukocytes

Білкові детермінанти мембран еритроцитів Protein determinants of erythrocyte membranes

123 / 200
Внаслідок травми у чоловіка 35-ти років настав повний розрив спинного мозку на рівні першого шийного сегменту. Як зміниться при цьому зовнішнє дихання? As a result of an injury, a 35-year-old man suffered a complete rupture of the spinal cord at the level of the first cervical segment. How will external breathing change?

Стане діафрагмальним Will become diaphragmatic

Не зміниться Will not change

Стане поверхневим і частим Will become shallow and frequent

Зупиниться Will stop

Стане рідким і глибоким It will become thin and deep

124 / 200
При обстеженні хворого на атрофічний гастрит виявлено мегалобла-стну анемію. Дефіцит якої речовини є причиною виникнення анемії у цього хворого? During the examination of a patient with atrophic gastritis, megaloblastic anemia was detected. The deficiency of which substance is the cause of anemia in this patient?

Гастромукопротеїд Gastromucoprotein

Вітамін Ві Vitamin Vi

Еритропоетини Erythropoietins

Вітамін В6 Vitamin B6

Залізо Iron

125 / 200
Хворому на сифіліс призначили лікарський засіб, в основі механізму дії якого лежить порушення утворення муреїну, що призводить до загибелі збудника. Визначить цей препарат: A syphilis patient was prescribed a drug whose mechanism of action is based on a violation of murein formation, which leads to the death of the pathogen. This drug will determine:

Азитроміцин Azithromycin

Ципрофлоксацин Ciprofloxacin

Бійохінол Bioquinol

Бензилпеніциліну натрієва сіль Benzylpenicillin sodium salt

Доксацикліну гідрохлорид Doxacycline hydrochloride

126 / 200
У хворого, що знаходиться у опіковому відділенні, виникло гнійне ускладнення. Гній, що виділяється, має синьо-зелений відтінок, що вказує на інфекцію, зумовлену Pseudomonas aeruginosa. Яка ознака характерна для цього збудника? A patient in the burn unit developed a purulent complication. The pus that is released has a blue-green tint, indicating an infection caused by Pseudomonas aeruginosa. What sign is characteristic of this pathogen?

Негативне забарвлення за Грамом Negative gram stain

Наявність спор Availability of disputes

Утворення міцелію Formation of mycelium

Розташування клітин парами Cell arrangement in pairs

Кокова форма Coco form

127 / 200
При забарвленні препарату з харкотиння хворого були використані наступні барвники та реактиви: розчин фуксину Циля, розчин метиленового синього, 5% розчин сірчаної кислоти. Який спосіб забарвлення було застосовано? The following dyes and reagents were used when staining the preparation from the patient's sputum: Ziel's fuchsin solution, methylene blue solution, 5% sulfuric acid solution. What staining method was used?

Циля-Нільсона Target-Nielson

Пєшкова Peshkova

Грама Gram

Гінса-Буррі Guinsa Burri

Нейссера Neissera

128 / 200
У людини цистинурія проявляється у вигляді наявності цистинових камінців у нирках (гомозиготи) або підвищеним рівнем цистину в сечі (гетерозиготи). Захворювання цистинурією є моногенним. Визначити тип взаємодії генів цистинурії і нормального вмісту цистину в сечі: In humans, cystinuria manifests itself as the presence of cystine stones in the kidneys (homozygotes) or an increased level of cystine in the urine (heterozygotes). The cystinuria disease is monogenic. Determine the type of gene interaction cystinuria and normal cystine content in urine:

Епістаз Epistasis

Кодомінування Codominance

Неповне домінування Incomplete dominance

Повне домінування Total dominance

Комплементарність Complementarity

129 / 200
При обстеженні в клініці у чоловіка діагностували гостру променеву хворобу. Лабораторно встановлено різке зниження серотоніну в тромбоцитах. Порушення метаболізму якої речовини є можливою причиною зниження тром-боцитарного серотоніну? During the examination in the clinic, the man was diagnosed with acute radiation sickness. A sharp decrease in serotonin in platelets was found in the laboratory. Disruption of the metabolism of which substance is a possible cause of the decrease in platelet serotonin?

5-окситриптофан 5-oxytryptophan

Фенілаланін Phenylalanine

Серин Serine

Гістидин Histidine

Тирозин Tyrosine

130 / 200
У хворої 40-ка років збільшена щитоподібна залоза. Під час пальпації залоза щільна, поверхня її дрібногорби-ста. При гістологічному дослідженні біоптату щитоподібної залози виявлено дифузну інфільтрацію тканини лімфоцитами, плазматичними клітинами, утворення лімфоїдних фолікулів. Яке захворювання у жінки? A 40-year-old patient has an enlarged thyroid gland. During palpation, the gland is dense, with small bumps on its surface. Histological examination of the thyroid gland biopsy revealed diffuse tissue infiltration by lymphocytes , plasma cells, the formation of lymphoid follicles. What disease does a woman have?

Тиреоїдит Ріделя Riedel's thyroiditis

Спорадичний зоб Sporadic goiter

Ендемічний зоб Endemic goiter

Дифузний токсичний зоб Diffuse toxic goiter

Аутоімунний тиреоїдит Autoimmune thyroiditis

131 / 200
Хворий потрапив до лікарні після іонізуючого опромінення зі скаргами на блювання, анорексію, біль у різних ділянках живота, наявність крові у калі, підвищення температури тіла, кволість. Для якої форми гострої променевої хвороби характерна клінічна карти- на? The patient was admitted to the hospital after ionizing radiation with complaints of vomiting, anorexia, pain in various parts of the abdomen, the presence of blood in the stool, increased body temperature, weakness. For which What is the characteristic clinical picture of a form of acute radiation sickness?

Кістковомозкова Bone marrow

Кишкова Intestinal

Змішана Mixed

Токсемічна Toxemic

Церебральна Cerebral

132 / 200
Хворому чоловіку 75-ти років, в якого частота серцевих скорочень була 40/хв, імплантували серцевий електростимулятор. Після цього ЧСС зросла до 70/хв. Функцію якого відділу серця взяв на себе електростимулятор? A 75-year-old man with a heart rate of 40/min was implanted with a pacemaker. After that, the heart rate increased to 70/min. The function of which department the heart was taken over by an electric stimulator?

Ніжки Гіса His Legs

Синоатрільного вузла Sino-atrial node

Волокон Пуркін’є Purkinje fiber

Волокон пучка Гіса His bundle fiber

Атріовентрикулярного вузла Atrioventricular node

133 / 200
У чоловіка 40-ка років на шиї виявлено пухлиноподібне утворення розміром 8х7 см, яке хірург видалив неповністю через інтимний зв’язок з великими судинами. Мікроскопічно в ньому виражений тканинний і клітинний атипізм, клітини типу ліпобластів різного ступеня зрілості, з поліморфізмом, гіперхромією ядер, патологічними мітозами, осередками некрозу. Визначте гістологічну форму пухлини: A 40-year-old man was found to have a tumor-like mass measuring 8x7 cm on his neck, which the surgeon removed incompletely due to its intimate connection with large vessels. Microscopically, it showed tissue and cellular atypism, lipoblast-type cells of varying degrees of maturity, with polymorphism, hyperchromia of nuclei, pathological mitoses, and foci of necrosis. Determine the histological form of the tumor:

Ліпосаркома Liposarcoma

Ліпома Lipoma

Фіброма Fibroma

Гібернома Hibernoma

Фібросаркома Fibrosarcoma

134 / 200
Відомо, що при цукровому діабеті у хворих частіше зустрічаються запальні процеси, знижена регенерація, уповільнюється загоєння ран. Причиною цього є: It is known that in patients with diabetes, inflammatory processes occur more often, regeneration is reduced, and wound healing slows down. The reason for this is:

Зниження ліполізу Decreased lipolysis

Підвищення ліполізу Increasing lipolysis

Посилення катаболізму Strengthening catabolism

Прискорення глюконеогенезу Acceleration of gluconeogenesis

Зниження протеосинтезу Reduction of proteosynthesis

135 / 200
У хворої 60-ти років знижена активність основного травного ферменту слини. В цьому випадку порушується первинний гідроліз: A 60-year-old patient has decreased activity of the main digestive enzyme of saliva. In this case, primary hydrolysis is disturbed:

Жирів Fat

Клітковини Fibre

Молочного цукру Milk sugar

Вуглеводів Carbs

Білків Proteins

136 / 200
Людина, що тривало приймала лі- ки, не може припинити їх використання, тому що при цьому у неї виникають порушення психічних і соматичних функцій. Як називається такий синдром, що виникає при відмові від прийому лікарської речовини? A person who has been taking drugs for a long time cannot stop using them, because at the same time he has disorders of mental and somatic functions. What is the name of such a syndrome, what happens when you stop taking the medicinal substance?

Ідіосинкразія Idiosyncrasy

Абстиненція Abstinence

Тахіфілаксія Tachyphylaxis

Сенсибілізація Sensitization

Кумуляція Cumulative

137 / 200
У жінки 49-ти років внаслідок тривалого стояння з’явився набряк ніг. Яка можлива причина появи набряків? A 49-year-old woman developed leg swelling as a result of standing for a long time. What is the possible cause of swelling?

Збільшення системного артеріального тиску Increase in systemic blood pressure

Збільшення онкотичного тиску плазми крові Increase in oncotic pressure of blood plasma

Зменшення гідростатичного тиску крові у венах Reduction of hydrostatic blood pressure in veins

Зменшення гідростатичного тиску крові в артеріях Reduction of hydrostatic blood pressure in arteries

Збільшення гідростатичного тиску крові у венах Increase in the hydrostatic pressure of the blood in the veins

138 / 200
Хворому після апендектомії з метою профілактики інфекції призначений антибіотик групи цефалоспоринів. Порушення якого процесу полягає в основі протимікробної активності антибіотиків цієї групи? After an appendectomy, a cephalosporin antibiotic was prescribed to the patient to prevent infection. The violation of which process is the basis of the antimicrobial activity of antibiotics of this group?

Енергетичний обмін Energy Exchange

Блокада холінестерази Cholinesterase blockade

Рибосомальний синтез білку Ribosomal protein synthesis

Синтез нуклеїнових кислот Synthesis of nucleic acids

Утворення мікробної стінки Microbial wall formation

139 / 200
Людина, що тривалий час знаходилася у задушливому приміщенні, знепритомніла. Свідомість відновилася після вдихання парів нашатирного спирту. З прямим впливом на які структури пов’язана дія цієї речовини? A person who was in a suffocating room for a long time fainted. Consciousness was restored after inhaling ammonia vapors. Which structures are directly affected by the action of this substance?

Судиноруховий центр Vascomotor Center

Дихальний центр Respiratory Center

Ємкісні судини Capacity vessels

Резистивні судини Resistive vessels

Рецептори верхніх дихальних шляхів Upper respiratory tract receptors

140 / 200
У хворого, що страждає на хронічну ниркову недостатність, виявлено у крові підвищення рівня залишкового азоту до 35 ммоль/л, більше половини якого складає сечовина. Виявлена гіпе-разотемія є: In a patient suffering from chronic renal failure, an increase in the level of residual nitrogen in the blood up to 35 mmol/l, more than half of which is urea. Hyperrazotemia was detected is:

Печінковою Hepatic

Ретенційною Retention

Резидуальною Residual

Продукційною Production

Змішаною Mixed

141 / 200
При огляді порожнини рота дитини педiатр знайшла 8 різців. Розвиток дитини відповідає віковій нормі. Визначте вік дитини: When examining the child's oral cavity, the pediatrician found 8 incisors. The child's development corresponds to the age norm. Determine the child's age:

16-20 місяців 16-20 months

12-15 місяців 12-15 months

10-12 місяців 10-12 months

6-7 місяців 6-7 months

7-8 місяців 7-8 months

142 / 200
У хворого на жовтяницю у крові виявлено збільшення загального білірубіну за рахунок непрямої його фракції. Сеча і кал інтенсивно забарвлені. Який найбільш вірогідний механізм вказаних порушень? In a patient with jaundice, an increase in total bilirubin was detected in the blood due to its indirect fraction. Urine and feces are intensely colored. What is the most likely mechanism of the specified disorders?

Утруднення відтоку жовчі з печінки Difficulty of outflow of bile from the liver

Порушення перетворення уробіліно-гену в печінці Disruption of urobilin gene conversion in the liver

Пошкодження паренхіми печінки Damage of liver parenchyma

Підвищений гемоліз еритроцитів Increased hemolysis of erythrocytes

Порушення утворення прямого білірубіну Disruption of direct bilirubin formation

143 / 200
При мікроскопічному дослідженні виявляється паренхіматозний орган, в якому епітеліальні тяжі формують клу-бочкову, пучкову та сітчасту зони. Центральна частина органу представлена скупченнями хромафінних клітин. Визначте орган: Microscopic examination reveals a parenchymal organ in which epithelial strands form lobular, bundle and reticular zones. The central part of the organ is represented by clusters of chromaffin cells. Identify the organ:

Епіфіз Pineal gland

Гіпофіз Pituitary

Щитоподібна залоза Thyroid

Печінка Liver

Надниркова залоза Adrenal gland

144 / 200
У хворого круглясті виразки на обличчі, запалення та збільшення лімфатичних вузлів. Ці симптоми з’явилися після укусів москітів. Під час лабораторного дослідження виділень із виразок на обличчі виявлено одноклітинні безджгутикові організми. Який діагноз найбільш вірогідний? The patient has round sores on his face, inflammation and enlarged lymph nodes. These symptoms appeared after mosquito bites. During the laboratory examination of secretions from the sores on his face, single-celled flagellate organisms. What is the most likely diagnosis?

Дерматотропний лейшманіоз Dermatotropic leishmaniasis

Короста Scabies

Трипаносомоз Trypanosomosis

Міаз Miaz

Токсоплазмоз Toxoplasmosis

145 / 200
У чоловіка має місце стеноз мі-трального отвору. Який механізм сер- цевої недостатності є провідним? A man has stenosis of the mitral orifice. What is the leading mechanism of heart failure?

Перевантаження опором Resistance overload

Пошкодження міокарда Myocardial damage

Перевантаження серця припливом крові Overload of the heart with blood flow

Перевантаження об’ємом Volume overload

Перевантаження напругою Voltage overload

146 / 200
Хворий проходив чергове обстеження, в результаті якого у нього виявлено гіперглікемію, кетонурію, поліурію, глюкозурію. Яка форма порушення КОС має місце за наявності цих явищ? The patient underwent another examination, as a result of which hyperglycemia, ketonuria, polyuria, glucosuria were detected. What form of KOS violation occurs in the presence of these phenomena?

Метаболічний ацидоз Metabolic acidosis

Газовий алкалоз Gas alkalosis

Метаболічний алкалоз Metabolic alkalosis

Газовий ацидоз Gas acidosis

Негазовий ацидоз Nongaseous acidosis

147 / 200
В препараті сполучної тканини дерми шкіри, забарвленому суданом-III і гематоксиліном, виявляються скупчення великих багатокутних клітин, які фарбуються в помаранчевий колір. Ядра мають сплощену форму, зміщені на периферію. Яка це тканина? In the preparation of the connective tissue of the skin dermis, stained with Sudan-III and hematoxylin, clusters of large polygonal cells are revealed, which are stained orange. The nuclei have a flattened shape, shifted to periphery. What tissue is this?

Біла жирова White fat

Бура жирова Bura fat

Гіалінова хрящова Hyaline cartilage

Ретикулярна сполучна Reticular connective

Пластинчаста кісткова Squamous bone

148 / 200
Введення знеболюючого пацієнту перед екстракцією зуба призвело до розвитку анафілактичного шоку, який супроводжувався розвитком олігурії. Який патогенетичний механізм зумовив зменшення діурезу в даній клінічній ситуації? The administration of an anesthetic to a patient before tooth extraction led to the development of anaphylactic shock, which was accompanied by the development of oliguria. What pathogenetic mechanism caused the decrease in diuresis in this clinical situation?

Збільшення онкотичного тиску плазми крові Increased blood plasma oncotic pressure

Пошкодження клубочкового фільтру Damage of the glomerular filter

Підвищення гідростатичного тиску в капсулі Шумлянського-Боумена Increase in hydrostatic pressure in the Shumlyansky-Bowman capsule

Зниження гідростатичного тиску в капілярах ниркових тілець Reduction of hydrostatic pressure in the capillaries of the renal corpuscles

Збільшення секреції вазопресину Increased vasopressin secretion

149 / 200
Хворого госпіталізовано з проникаючою травмою дна порожнини рота. Який м’яз травмований? The patient was hospitalized with a penetrating injury to the floor of the oral cavity. What muscle is injured?

Щито -під’язиковий Shield - sublingual

Груднинно-під’язиковий Maternal-hyoid

Щелепно-під’язиковий Maxillohyoid

Шило-під’язиковий Silohyoid

Лопатково-під’язиковий Scapulohyoid

150 / 200
У хворого з переломом стегнової кістки в ділянці хірургічної шийки з’явились ознаки гострої правошлуночко-вої недостатності внаслідок емболії легеневої артерії. Який вид емболії має місце? A patient with a femur fracture in the area of the surgical neck developed signs of acute right ventricular failure due to pulmonary embolism. What type of embolism is there?

Метастатична Metastatic

Жирова Fat

Газова Gas

Повітряна Air

Тканинна Fabric

151 / 200
Внаслідок вивиху нижньої щелепи у пацієнта порушена смакова чутливість передніх 2/3 язика та слиновиділення. Травмою якого нерва це викликано? Due to the dislocation of the lower jaw, the patient has impaired taste sensitivity of the front 2/3 of the tongue and salivation. What nerve injury is this caused by?

Великий кам’янистий нерв Great stony nerve

Під’язиковий нерв Hypoglossal nerve

Малий кам’янистий нерв Small stony nerve

Барабанна струна Drum String

Глибокий кам’янистий нерв Deep stony nerve

152 / 200
Пацієнт скаржиться на біль у верхній щелепі та зубах. Об’єктивно: відмічається біль при натискуванні в ділянці підочноямкового отвору. Який нерв уражено? The patient complains of pain in the upper jaw and teeth. Objectively: pain is noted when pressing in the area of the infraorbital opening. Which nerve is affected?

Блоковий нерв Block nerve

Перша гілка трійчастого нерва First branch of the trigeminal nerve

Третя гілка трійчастого нерва Third branch of the trigeminal nerve

Друга гілка трійчастого нерва Second branch of the trigeminal nerve

Лицевий нерв Facial nerve

153 / 200
Хворий 40-ка років помер від набряку головного мозку. В анамнезі карбункул обличчя. На аутопсії відмічено повнокров’я та набряк тканини головного мозку. У білій речовині лівої півкулі виявлено дві порожнини розмірами 6х5,5 см та 5х4,5 см, що заповнені вершковоподібною рідиною жовтувато-зеленого кольору. Стінками порожнин є нервова тканина з нерівними краями. Яке ускладнення карбункула розвинулось у хворого? A 40-year-old patient died of cerebral edema. He has a history of facial carbuncles. At autopsy, hemorrhaging and swelling of the brain tissue were noted. In the white matter of the left hemisphere two cavities measuring 6x5.5 cm and 5x4.5 cm were found, filled with a yellowish-green creamy liquid. The walls of the cavities are nerve tissue with uneven edges. What complication of the carbuncle developed in the patient?

Колікваційні некрози Collicative necrosis

Кісти Cysts

Хронічні абсцеси Chronic abscesses

Гострі абсцеси Acute abscesses

Емпієма Empyema

154 / 200
При дослідженні трубчастого органу встановлено, що його середня оболонка складається із гіалінових суцільних кілець. Який епітелій вистилає слизову оболонку цього органа? When examining a tubular organ, it was established that its middle membrane consists of hyaline continuous rings. What epithelium lines the mucous membrane of this organ?

Одношаровий кубічний One layer cubic

Багатошаровий плоский незрогові-лий Multilayer flat non-horny

Одношаровий призматичний залозистий Single-layer prismatic glandular

Багаторядний призматичний війчастий Multi-row prismatic ciliate

Одношаровий призматичний з облямівкою Single-layer prismatic with a border

155 / 200
Хвора госпіталізована з діагнозом плеврит. В якому місці плевральної порожнини міститься найбільша кількість ексудату? The patient was hospitalized with a diagnosis of pleurisy. In which part of the pleural cavity is the largest amount of exudate?

Реберно-медіастинальний синус Cost-mediastinal sinus

Під коренем легенів Under the root of the lungs

Під куполом плеври Under the dome of the pleura

Реберно-діафрагмальний синус Cost-phrenic sinus

Діафрагмально-медіастинальний синус Diaphragmatic-mediastinal sinus

156 / 200
При дослідженні бронхобіоптату встановлено: атрофія слизової оболонки, кістозне перетворення залоз, осередкова метаплазія покривного призматичного епітелію в багатошаровий плоский, збільшення числа келихоподібних клітин; місцями - у стінці бронха та особливо у слизовій оболонці різко виражена клітинна запальна інфільтрація і розростання грануляційної тканини, яка вибухає у просвіт бронха у вигляді поліпа. Який найбільш імовірний діагноз? During examination of the bronchobioptate, it was found: atrophy of the mucous membrane, cystic transformation of the glands, focal metaplasia of the covering prismatic epithelium into a multilayered flat one, an increase in the number of goblet cells; in places - in the wall of the bronchus and especially in the mucous membrane, cellular inflammatory infiltration and growth of granulation tissue, which explodes into the lumen of the bronchus in the form of a polyp, is sharply expressed. What is the most likely diagnosis?

Гострий бронхіт Acute bronchitis

Часткова пневмонія Partial pneumonia

Хронічний бронхіт Chronic bronchitis

Бронхопневмонія Bronchopneumonia

Інтерстиційна пневмонія Interstitial pneumonia

157 / 200
У місцевому імунітеті порожнини рота важливу роль відіграють різноманітні клітини слизової оболонки і антимікробні речовини, які синтезуються ними. Які з перерахованих факторів мають вирішальну роль у забезпеченні локального імунітету? In the local immunity of the oral cavity, various cells of the mucous membrane and the antimicrobial substances synthesized by them play an important role. Which of the listed factors play a decisive role in providing local immunity?

Еозинофіли Eosinophils

IgG IgG

Макрофаги Macrophages

В-лімфоцити B-lymphocytes

Секреторний IgA Secretory IgA

158 / 200
На прийомі лікар-стоматолог виявив у жінки 36-ти років на щічній поверхні ясни у ділянці 2-го моляра утворення у вигляді вузлика діаметром 0,8 см темно-бурого кольору, м’якої консистенції, на широкій основі. Гістологічно утворення багате на судини си- нусоїдного типу з великою кількістю округлих одноядерних і великих багатоядерних клітин, місцями зустрічається скупчення гранул гемосидерину. Який найбільш імовірний діагноз? During the appointment, the dentist discovered in a 36-year-old woman on the buccal surface of the gums in the area of the 2nd molar a formation in the form of a nodule with a diameter of 0.8 cm dark brown color, soft consistency, on a wide base. Histologically, the formation is rich in vessels of the sinusoidal type with a large number of rounded mononuclear and large multinucleated cells, in places there is an accumulation of hemosiderin granules. What is the most likely diagnosis?

Остеобластокластома щелепи Osteoblastoclastoma of the jaw

Ангіоматозний епулід Angiomatous epulid

Гігантоклітинний епулід Giant cell epulid

Прикоренева гранульома Principal granuloma

Амелобластома Ameloblastoma

159 / 200
Для знечулення різців верхньої щелепи анестетик вводять в ділянку різцевого отвору. Який нерв залягає в цьому місці? To anesthetize the incisors of the upper jaw, an anesthetic is injected into the area of the incisor opening. What nerve lies in this place?

Nn.palatini minores Nn.palatini minores

N.palatinus major N. palatinus major

Rr.nasales posteriores inferiores Rr.nasales posteriores inferiores

N.nasopalatinus N.nasopalatinus

N.pharyngeus N.pharyngeus

160 / 200
В процесі гістогенезу тканин зуба з певних причин вчасно не утворився дентин. Який процес подальшого гістогенезу не відбудеться або буде віддалений у часі? In the process of histogenesis of tooth tissues, dentin was not formed in time for certain reasons. Which process of further histogenesis will not take place or will be distant in time?

Утворення предентинового простору Formation of predentine space

Утворення пульпи Pulp Formation

Утворення клітинного цементу Formation of cellular cementum

Утворення емалі Enamel formation

Утворення безклітинного цементу Acellular cement formation

161 / 200
При огляді порожнини носа виявлено викривлення задньої частини носової перегородки. Яка кістка викривлена? When examining the nasal cavity, a curvature of the back part of the nasal septum was found. Which bone is curved?

Леміш Lemish

Вертикальна пластинка піднебінної кістки Vertical plate of the palatine bone

Медіальна пластинка крилоподібного відростка Medial plate of pterygium

Латеральна пластинка крилоподібного відростка Lateral plate of pterygium

Перпендикулярна пластинка решітчастої кістки Perpendicular ethmoid plate

162 / 200
У чоловіка 70-ти років розвинувся протезний стоматит. Крім того, спостерігалось виражене ураження куточків рота. Під час мікроскопії виявлені великі овоїдні грампозитивні клітини. Які мікроорганізми найбільш імовірно стали провідним етіологічним фактором такого ураження? A 70-year-old man developed prosthetic stomatitis. In addition, severe damage to the corners of the mouth was observed. During microscopy, large ovoid gram-positive cells were detected. Which microorganisms are most likely to have become the leading etiological factor of such a lesion?

Коринебактерії Corynebacteria

Гриби роду Candida Candida fungi

Стафілококи Staphylococci

Стрептококи Streptococci

Нейсерії Neisseria

163 / 200
При остеолатеризмі зменшується міцність колагену, зумовлена помітним зменшенням утворення поперечних зшивок у колагенових фібрилах. Причиною цього є зниження активності: With osteolaterism, the strength of collagen decreases due to a noticeable decrease in the formation of cross-links in collagen fibrils. The reason for this is a decrease in activity:

Моноаміноксидази Monoamine oxidases

Лізилгідроксилази Lysylhydroxylases

Пролілгідроксилази Prolyl hydroxylases

Лізилоксидази Lysyl oxidases

Колагенази Collagenases

164 / 200
У біоптаті нирки виявлено: проміжна тканина інфільтрована лейкоцитами, міліарні абсцеси, канальці в стані дистрофії, заповнені десквамованим епітелієм та лейкоцитами. Про яке захворювання можна думати? In a kidney biopsy, it was found: interstitial tissue infiltrated with leukocytes, miliary abscesses, tubules in a state of dystrophy, filled with desquamated epithelium and leukocytes. What kind of disease can you think about?

Пієлонефрит Pyelonephritis

Нефролітіаз Nephrolithiasis

Гломерулонефрит Glomerulonephritis

Пієліт Pyelitis

Некротичний нефроз Necrotic nephrosis

165 / 200
У дитини з нирковою недостатністю виявлена затримка прорізування зубів. Порушення утворення в нирках якої речовини найбільш імовірно є причиною цього? A child with kidney failure is found to have delayed teething. Disturbance in the formation of which substance in the kidneys is most likely the cause of this?

а-кетоглутарат α-ketoglutarate

1,25 (OH)2D3 1.25 (OH)2D3

Гідроксильований лізин Hydroxylated Lysine

Глутамат Glutamate

Ілікоціамін Ilychocyamine

166 / 200
У чоловіка під час операції з приводу пахвинної грижі хірург пошкодив вміст пахвинного каналу. Яку структуру пошкодив хірург? In a man, during an operation for an inguinal hernia, the surgeon damaged the contents of the inguinal canal. What structure did the surgeon damage?

Funiculus spermaticus Funiculus spermaticus

- -

Lig. inguinale Lig. inguinale

Urarchus Urarchus

Lig. teres uteri Lig. teres uteri

167 / 200
Чоловік 26-ти років внаслідок автомобільної аварії перебуває в торпідній стадії шоку. У крові: лейк.- 3, 2 • 109/л. Який головний механізм в розвитку лейкопенії? A 26-year-old man is in the torpid stage of shock as a result of a car accident. In the blood: leuk.- 3, 2 • 109/l. What is the main mechanism in the development leukopenia?

Руйнування лейкоцитів у кровотворних органах Destruction of leukocytes in hematopoietic organs

Перерозподіл лейкоцитів у судинному руслі Redistribution of leukocytes in the vascular bed

Порушення виходу зрілих лейкоцитів з кісткового мозку в кров Disturbance of release of mature leukocytes from the bone marrow into the blood

Пригнічення лейкопоезу Suppression of leukopoiesis

Підвищення виділення лейкоцитів з організму Increased release of leukocytes from the body

168 / 200
Під час дослідження чоловіка 24-х років виявлені наступні зміни сечі: добовий діурез - 10 літрів, відносна густина сечі -1,001, якісні зміни відсутні. Пацієнт скаржиться на сильну спрагу, часте сечовиділення. Що є найбільш імовірною причиною даного захворювання? During the examination of a 24-year-old man, the following changes in urine were detected: daily diuresis - 10 liters, relative density of urine -1.001, no qualitative changes. The patient complains of strong thirst, frequent urination. What is the most likely cause of this disease?

Гіперсекреція альдостерону Hypersecretion of aldosterone

Гіперсекреція глюкокортикоїдів Hypersecretion of glucocorticoids

Гіпосекреція вазопресину Vasopressin hyposecretion

Гіперсекреція вазопресину Hypersecretion of vasopressin

Відносна інсулінова недостатність Relative insulin deficiency

169 / 200
При регенерації епітелію слизової оболонки порожнини рота (розмноження клітин) відбулася реплікація (авторепродукція) ДНК за напівконсерва-тивним механізмом. При цьому нукле-отиди нової нитки ДНК є комплементарними до: During the regeneration of the epithelium of the mucous membrane of the oral cavity (cell reproduction), replication (autoreproduction) of DNA took place according to a semi-conservative mechanism. At the same time, the nucleotides of the new DNA strand are complementary to:

Материнської нитки Mother threads

Ферменту РНК-полімерази RNA polymerase enzyme

Ферменту ДНК-полімерази DNA polymerase enzyme

Змістовних кодонів Content codons

!нтронних ділянок гену !ntronic regions of the gene

170 / 200
У дитини 7-ми років, хворої на ангіну, був взятий мазок з поверхні мигдаликів і засіяний на кров’яний агар. Через добу виросли колонії стрептококів, навколо яких середовище стало прозорим. Наявність якого фактора патогенності у збудника виявило це дослідження? A 7-year-old child with tonsillitis had a smear taken from the surface of the tonsils and inoculated on blood agar. A day later, colonies of streptococci grew, around which the environment became transparent. The presence of which factor of pathogenicity in the causative agent was detected by this study?

Нейрамінідаза Neuraminidase

Бета -лактамаза Beta-lactamase

Лейкоцидин Leukocidin

Ендотоксин Endotoxin

Гемолізин Hemolysin

171 / 200
Хлопчик 5-ти місяців госпіталізований з приводу тонічних судом. Хворіє з народження. При огляді волосся жорстке, нігті витончені та ламкі, шкірні покриви бліді та сухі. У крові: кальцій -1,5 ммоль/л, фосфор -1,9 ммоль/л. З чим пов’язані ці зміни? A 5-month-old boy is hospitalized due to tonic convulsions. He has been sick since birth. On examination, the hair is hard, the nails are thin and brittle, the skin is pale and dry. In the blood : calcium -1.5 mmol/l, phosphorus -1.9 mmol/l. What are these changes related to?

Гіпотиреоз Hypothyroidism

Гіпоальдостеронізм Hypoaldosteronism

Гіпопаратиреоз Hypoparathyroidism

Гіперальдостеронізм Hyperaldosteronism

Гіперпаратиреоз Hyperparathyroidism

172 / 200
У чоловіка 60-ти років спостерігається послаблення перистальтики кишечнику. Який з перерахованих харчових продуктів буде стимулювати перистальтику в найбільшій мірі? A 60-year-old man has a weakening of intestinal peristalsis. Which of the listed food products will stimulate peristalsis to the greatest extent?

М’ясо Meat

Сало Lard

Білий хліб White bread

Чорний хліб Black bread

Чай Tea

173 / 200
При пункційній біопсії печінки хворого з клінікою печінково-клітинної недостатності виявлено вакуольну, балонну дистрофію гепатоцитів, некроз окремих клітин, тільця Каунсильмена, інфільтрацію портальної та часточкової строми переважно лімфоцитами, макрофагами з незначною кількістю поліморфноядерних лейкоцитів. Який найбільш імовірний діагноз? A puncture biopsy of the liver of a patient with a clinic of hepatocellular insufficiency revealed vacuolar, ballooning dystrophy of hepatocytes, necrosis of individual cells, Kaunsilman bodies, infiltration of the portal and lobular stroma mainly by lymphocytes, macrophages with a small number of polymorphonuclear leukocytes. What is the most likely diagnosis?

Хронічний активний гепатит Chronic active hepatitis

Хронічний персистуючий гепатит Chronic persistent hepatitis

Гострий вірусний гепатит Acute viral hepatitis

Аутоімунний гепатит Autoimmune hepatitis

Алкогольний гепатит Alcoholic hepatitis

174 / 200
При посіві матеріалу із зіва хворого ангіною на кров’яно-телуритовий агар виросли колонії діаметром 4-5 мм, сірого кольору, радіально покреслені (у вигляді розеток). Під мікроскопом грам-позитивні палички із булавоподібними потовщеннями на кінцях, розміщені у вигляді розчепірених пальців. Які це мікроорганізми? When inoculating the material from the pharynx of a angina patient on blood-tellurite agar, colonies with a diameter of 4-5 mm, gray color, radially outlined (in the form of rosettes) grew. Under the microscope, gram-positive rods with club-like thickenings at the ends, placed in the form of splayed fingers. What are these microorganisms?

Клостридії ботулізму Clostridia botulism

Стрептококи Streptococci

Дифтероїди Diphtheroids

Коринебактерії дифтерії Diphtheria corynebacteria

Стрептобацили Streptobacilli

175 / 200
Під час цитогенетичного обстеження пацієнта з порушеною репродуктивною функцією виявлено в деяких клітинах нормальний каріотип 46,ХУ але у більшості клітин каріотип синдрому Клайнфельтера - 47,ХХУ. Яку назву носить таке явище неоднорідності клітин? During the cytogenetic examination of a patient with impaired reproductive function, a normal karyotype of 46,XU was found in some cells, but in most cells the karyotype of Klinefelter syndrome was 47,XXU. What is the name of such a phenomenon of cell heterogeneity?

Мономорфізм Monomorphism

Транспозиція Transposition

Інверсія Inversion

Мозаїцизм Mosaicism

Дуплікація Duplication

176 / 200
При диспансерному обстеженні хлопчику 7-ми років встановлено діагноз - синдром ЛешаНайхана (хворіють тільки хлопчики). Батьки здорові, але у дідуся за материнською лінією таке ж захворювання. Який тип успадкування захворювання? During a dispensary examination, a 7-year-old boy was diagnosed with Lesha Naihan syndrome (only boys are affected). The parents are healthy, but the maternal grandfather has the same disease. What type of disease inheritance?

Автосомно-рецесивний Autosomal recessive

Домінантний, зчеплений із статтю Sex-linked dominant

Рецесивний, зчеплений із статтю Sex-linked recessive

Автосомно-домінантний Autosomal dominant

Неповне домінування Incomplete dominance

177 / 200
В експерименті необхідно оцінити рівень збудливості клітини. Для цього доцільно визначити: In the experiment, it is necessary to estimate the level of cell excitability. For this, it is advisable to determine:

Потенціал спокою Rest potential

Критичний рівень деполяризації Critical level of depolarization

Амплітуду ПД Amplitude of PD

Поріг деполяризації Depolarization Threshold

Тривалість ПД Duration of PD

178 / 200
В кардіологічному відділенні в хворого виникла аритмія. Лікар призначив аміодарон. Який основний механізм протиаритмічної дії аміодарону? In the cardiology department, the patient developed an arrhythmia. The doctor prescribed amiodarone. What is the main mechanism of the antiarrhythmic action of amiodarone?

Активує серотонінові рецептори Activates serotonin receptors

Пригнічує холінорецептори Suppresses cholinergic receptors

Змінює чутливість міокарду до ацетилхоліну Changes the sensitivity of the myocardium to acetylcholine

Стимулює гістамінові рецептори Stimulates histamine receptors

Переважно блокує калієві канали Predominantly blocks potassium channels

179 / 200
При знеболенні слизової оболонки порожнини рота у пацієнта виник анафілактичний шок (генералізована ва-зодилятація, збільшення судинної проникності із виходом рідини із судин у тканини). Гіперчутливість якого типу розвинулася у пацієнта? When anesthetizing the mucous membrane of the oral cavity, the patient developed anaphylactic shock (generalized vasodilatation, increased vascular permeability with the release of fluid from the vessels into the tissue). What type of hypersensitivity developed in the patient?

І типу (анафілактична) Type I (anaphylactic)

IV типу (клітинна цитотоксичність) Type IV (cellular cytotoxicity)

ІІІ типу (імунокомплексна) Type III (immunocomplex)

V типу (гранульоматоз) Type V (granulomatosis)

ІІ типу (антитілозалежна) Type II (antibody dependent)

180 / 200
В експерименті у тварини перерізали таламокортикальні шляхи. Який вид сенсорних відчуттів у піддослідної тварини зберігся? In the experiment, the animal's thalamocortical pathways were cut. What kind of sensory sensations was preserved in the experimental animal?

Нюхові Olfactory

Слухові Aural

Ноцицептивні Nociceptive

Зорові Visual

Екстероцептивні Exteroceptive

181 / 200
Хворому з набряками призначено K+-зберігаючий діуретик - антагоніст альдостерону. Визначте цей препарат: A patient with edema is prescribed a K+-sparing diuretic - an aldosterone antagonist. Identify this drug:

Клофелін Clofelin

Дигоксин Digoxin

Алопуринол Allopurinol

Спіронолактон Spironolactone

Новокаїнамід Novocaineamide

182 / 200
У хворих на еритропоетичну пор-фірію (хвороба Гюнтера) зуби флюо-ресцують в ультрафіолеті яскраво червоним кольором, шкіра чутлива до світла, сеча забарвлена в червоний колір. З недостатністю якого ферменту пов’язана ця хвороба? In patients with erythropoietic porphyria (Gunter's disease), the teeth fluoresce bright red in ultraviolet light, the skin is sensitive to light, and the urine is colored red. What enzyme deficiency is associated with this disease?

Дельта-амінолевулінатсинтаза Delta-aminolevulinate synthase

Уропорфіриноген-ІІІ-косинтаза Uroporphyrinogen III cosynthase

Феррохелатаза Ferrochelatase

Уропорфіриноген-декарбоксилаза Uroporphyrinogen decarboxylase

Уропорфіриноген-І-синтаза Uroporphyrinogen-I synthase

183 / 200
Хворому 25-ти років з клінічною картиною нефротичного синдрому проведено пункційну біопсію нирки. Під час мікроскопічного дослідження клітини епітелію проксимальних канальців нефрону збільшені в об’ємі, у цитоплазмі вакуолі з прозорою рідиною, ядро зміщене до периферії. Яка дистрофія виявлена в епітелії каналь-ців? A 25-year-old patient with a clinical picture of nephrotic syndrome underwent a puncture biopsy of the kidney. During a microscopic examination, the cells of the epithelium of the proximal tubules of the nephron are enlarged in volume, in the cytoplasm of vacuoles with a clear liquid, the nucleus is shifted to the periphery. What kind of dystrophy is found in the epithelium of the tubules?

Гідропічна Hydropic

Зерниста Granular

Жирова Fat

Рогова Rohova

Гіаліново-крапельна Hyaline-drip

184 / 200
До стоматолога звернувся хворий з ураженням слизової оболонки ротової порожнини. Було встановлено діагноз: герпетичний стоматит. Який з перелічених засобів забезпечить дію на етіо-тропний фактор? A patient with lesions of the mucous membrane of the oral cavity consulted a dentist. The diagnosis was established: herpetic stomatitis. Which of the listed remedies will provide an effect on the etiotropic factor?

Парацетамол Paracetamol

Димедрол Diphenhydramine

Ацикловір Acyclovir

Левамізол Levamizole

Фурацилін Furacilin

185 / 200
У пацієнта 42-х років, що страждає на парадонтоз, у коронковій частині пульпи виявлені округлі звапно- вані утворення діаметром 2-3 мм. Назвіть ці структури: In a 42-year-old patient suffering from periodontitis, rounded calcified formations with a diameter of 2-3 mm were found in the crown part of the pulp. Name these structures:

!нтерглобулярний дентин !interglobular dentin

!нтерглобулярні простори !interglobular spaces

Склерозований (прозорий) дентин Sclerosed (transparent) dentine

Дєнтиклі Denticles

Мертвий дентин Dead dentin

186 / 200
У хворого виявлено порушення секреторної функції піднижньощелепної слинної залози. Який нерв забезпечує її вегетативну іннервацію? The patient has a violation of the secretory function of the submandibular salivary gland. What nerve provides its vegetative innervation?

N.petrosus major N.petrosus major

Chorda tympani Chorda tympani

N.auriculotemporalis N.auriculotemporalis

N.petrosus minor N.petrosus minor

N.mandibularis N.mandibularis

187 / 200
При гістологічному дослідженні мі-кропрепарату злоякісної пухлини легень виявлено, що вона складається із лімфоцитоподібних клітин, що не утворюють будь-яких структур. Строма виражена мало, спостерігається багато мітозів та некрозів. Яка це пухлина? During a histological examination of a micro-preparation of a malignant lung tumor, it was found that it consists of lymphocyte-like cells that do not form any structures. The stroma is poorly expressed, many mitoses are observed and necrosis. What kind of tumor is this?

Фіброма Fibroma

Аденокарцинома Adenocarcinoma

Плоскоклітинний незроговілий рак Squamous nonkeratinous carcinoma

Дрібноклітинний рак Small cell cancer

Плоскоклітинний зроговілий рак Squamous keratinized cancer

188 / 200
До стоматолога звернувся чоловік 35-ти років зі скаргами на зменшення щільності зубної тканини, підвищену крихкість зубів при вживанні твердої їжі. Лабораторно визначили співвідношення Ca/P в емалі при зіскоблюванні. Яке значення цього показника свідчить про посилення демінералізації? A 35-year-old man came to the dentist with complaints of a decrease in the density of dental tissue, increased fragility of teeth when eating solid food. The laboratory determined the Ca/P ratio in enamel at scraping. What value of this indicator indicates the strengthening of demineralization?

1,85 1.85

0,9 0.9

2,5 2.5

1,67 1.67

1,5 1.5

189 / 200
У пацієнта вилучений верхній медіальний різець. Гілки якої артерії його кровопостачають? The patient has had his upper medial incisor removed. Which artery's branches supply him with blood?

A.sphenopalatina A.sphenopalatina

A.infraorbitalis A.infraorbitalis

A.palatina descendens A.palatina descendens

A.alveolaris inferior A.alveolaris inferior

A.buccalis A.buccalis

190 / 200
У хворої 35-ти років з хронічним періодонтитом біля кореня 15 зуба видалено кісту діаметром 3 см. При гістологічному досліджені встановлено, що стінка її тонка, утворена зрілою сполучною тканиною, яка інфільтрована лімфоцитами і плазматичними клітинами, внутрішня поверхня вистелена багатошаровим плоским епітелієм без ознак кератинізації, в порожнині серозний ексудат. Який найбільш імовірний діагноз? A 3-cm-diameter cyst was removed from a 35-year-old patient with chronic periodontitis near the root of tooth 15. Histological examination revealed that its wall is thin, formed by mature connective tissue , which is infiltrated with lymphocytes and plasma cells, the inner surface is lined with multilayered flat epithelium without signs of keratinization, there is serous exudate in the cavity. What is the most likely diagnosis?

Фолікулярна амелобластома Follicular ameloblastoma

Херувізм Cherubism

Радикулярна кіста Radicular cyst

Примордіальна кіста Primordial cyst

Фолікулярна кіста Follicular cyst

191 / 200
При гістологічному дослідженні міокарда у хворого 47-ми років з ревматичною вадою серця (секційний матеріал) у кардіоміоцитах виявлені великі оптично порожні вакуолі. При забарвленні осмієвою кислотою вони чорного кольору, при забарвленні суданом III - жовто-червоні. Назвіть вид патологічного процесу: During histological examination of the myocardium of a 47-year-old patient with rheumatic heart disease (sectional material), large optically empty vacuoles were found in cardiomyocytes. When stained with osmic acid, they are black in color , when stained with Sudan III - yellow-red. Name the type of pathological process:

Гіаліново-крапельна дистрофія Hyaline-droplet dystrophy

Жирова дистрофія Fat dystrophy

Диспротеїноз Dysproteinosis

Вуглеводна дистрофія Carbohydrate dystrophy

Гідропічна дистрофія Hydropic dystrophy

192 / 200
Чутливий нервовий ганглій складається з нейроцитів кулястої форми з одним відростком, який на певній відстані від перикаріону поділяється на аксон і дендрит. Як називаються такі клітини? A sensitive nerve ganglion consists of spherical neurocytes with one process, which at a certain distance from the perikaryon divides into an axon and a dendrite. What are such cells called?

уніполярні unipolar

аполярні apolar

мультиполярні multipolar

псевдоуніполярні pseudounipolar

біполярні bipolar

193 / 200
У пацієнта навесні з’явилися пете-хіальні крововиливи, розхитування зубів, він відмічає високу чутливість до простудних хвороб. Лікар припустив гіповітаміноз C. Чим пояснюються зміни з боку зубів? In the spring, the patient developed petechial hemorrhages, loose teeth, he notes a high sensitivity to colds. The doctor assumed hypovitaminosis C. What explains the changes in the teeth ?

Механічне ушкодження зубів Mechanical damage to teeth

Порушення структури колагену періодонтальних зв’язок Disruption of the collagen structure of periodontal ligaments

Зміна структури глікозаміногліканів Change in structure of glycosaminoglycans

Підвищення проникності мембран навколозубних тканин Increasing permeability of membranes of peri-dental tissues

Порушення окисно-відновних процесів у навколозубних тканинах Disruption of oxidation-reduction processes in peri-dental tissues

194 / 200
До лабораторії особливо небезпечних інфекцій доставлено матеріал хворого з підозрою на холеру. Який ме- тод експрес діагностики може підтвердити цей діагноз? The material of a patient with suspicion of cholera was delivered to the laboratory of especially dangerous infections. What method of rapid diagnosis can confirm this diagnosis?

РГА РГА

PЗК PZK

РП RP

РА RA

РІФ RIF

195 / 200
При обстеженні хворого встановлено, що причиною гіпоплазії зу6ів є гіповітаміноз A та D. Ці вітаміни призначили перорально, проте лікувального ефекту не досягли. Яка можлива причина порушення засвоєння вітамінів? During the examination of the patient, it was established that hypovitaminosis A and D is the cause of the hypoplasia of the teeth. These vitamins were prescribed orally, but they did not achieve a therapeutic effect. What is the possible cause of impaired absorption of vitamins?

Ахлоргідрія Achlorhydria

Нестача жовчних кислот Lack of bile acids

Ахілія Achillia

Гіпохлоргідрія Hypochlorhydria

Гіперхлоргідрія Hyperchlorhydria

196 / 200
У хворого на туберкульоз застосовували рифампіцин, що призвело до розвитку стійкості до препарату мікобактерій туберкульозу. З яким препаратом необхідно поєднувати рифампіцин для зменшення розвитку стійкості мікобактерій? Rifampicin was used in a patient with tuberculosis, which led to the development of resistance to the drug of tuberculosis mycobacteria. With which drug should rifampicin be combined to reduce the development of resistance of mycobacteria?

Ацикловір Acyclovir

Амоксицилін Amoxicillin

!нтраконазол !ntraconazole

Ьоніазид Oniazid

Метронідазол Metronidazole

197 / 200
До стоматолога звернувся хворий з скаргами на біль, почервоніння, припухлість ясен. Запідозрено герпетичний гінгівостоматит. Який вірус міг викликати це захворювання? A patient came to the dentist with complaints of pain, redness, swelling of the gums. Herpetic gingivostomatitis is suspected. What virus could have caused this disease?

Вірус цитомегалії Cytomegaly virus

Вірус оперізуючого лишаю Zingles virus

Вірус простого герпеса тип 1 Herpes simplex virus type 1

Вірус простого герпеса тип 2 Herpes simplex virus type 2

Вірус Епштейн-Бара Epstein-Barr virus

198 / 200
Під час розтину тіла жінки 35-ти років поряд із збільшенням багатьох лімфатичних вузлів була виявлена збільшена селезінка вагою 600,0 г; на розрізі неоднорідна, темночервоного кольору, щільна, з ділянками некрозу сірувато-жовтуватого кольору, в діаметрі до 1 см (порфірова селезінка). Про яке захворювання можна думати? During the autopsy of the body of a 35-year-old woman, an enlarged spleen weighing 600.0 g was found along with an increase in many lymph nodes; on cross-section, it is heterogeneous, dark red in color, dense , with areas of grayish-yellow necrosis, up to 1 cm in diameter (porphyritic spleen). What disease can you think of?

Метастази раку Cancer metastases

Лімфосаркома Lymphosarcoma

Хронічний мієлоїдний лейкоз Chronic myeloid leukemia

Хронічний лімфоїдний лейкоз Chronic lymphoid leukemia

Лімфогранулематоз Lymphogranulomatosis

199 / 200
Хворому на стоматит лікар призначив полоскання ротової порожнини. Який антисептик з групи окислювачів найбільш придатний для цього? A patient with stomatitis was prescribed a mouth rinse by the doctor. Which antiseptic from the group of oxidizers is most suitable for this?

Розчин йоду спиртовий Iodine alcohol solution

Хлорамін Chloramine

Борна кислота Boric acid

Спирт етиловий Ethyl alcohol

Калію перманганат Potassium permanganate

200 / 200
При огляді порожнини рота чоловіка 60-ти років виявлені наступні зміни: 26 та 27 вкриті металічними коронками, які глибоко заходять під ясна. Між ними пародонтальна кишеня глибиною 0,7 см з незначною кількістю гною. Ясенні сосочки цих зубів гіпере-мовані, набряклі, з ціанотичним відтінком, при дотику зондом кровоточать. На рентгенограмі - резорбція міжзубних перегородок на 1/2 довжини кореня. Який найбільш вірогідний діагноз? When examining the oral cavity of a 60-year-old man, the following changes were detected: 26 and 27 are covered with metal crowns that go deep under the gums. Between them is a periodontal pocket with a depth of 0, 7 cm with a small amount of pus. The gingival papillae of these teeth are hyperemic, swollen, with a cyanotic hue, and bleed when touched with a probe. The radiograph shows resorption of the interdental septa for 1/2 the length of the root. What is the most likely diagnosis?

Локальний пародонтит Local periodontitis

Генералізований пародонтит Generalized periodontitis

Хронічний катаральний гінгівіт Chronic catarrhal gingivitis

- -

Гіпертрофічний гінгівіт Hypertrophic gingivitis